Download as pdf or txt
Download as pdf or txt
You are on page 1of 50

PrepU Chapter 52: Endocrine (Exam 1)

Study online at https://quizlet.com/_618nao


A client with hyperparathyroidism declines surgery and is to re-
-"Maintain a moderate exercise program."
ceive hormone replacement therapy with estrogen and proges-
The nurse should instruct the client to maintain a moderate ex-
terone. Which instruction is most important for the nurse to include
ercise program. Such a program helps strengthen bones and
in the client's teaching plan?
prevents the bone loss that occurs from excess parathyroid hor-
mone. Walking or swimming provides the most beneficial exercise.
-"Maintain a moderate exercise program."
Because of weakened bones, a rigorous exercise program such
-"Rest as much as possible."
as jogging is contraindicated. Weight loss might be beneficial but
-"Lose weight."
it isn't as important as developing a moderate exercise program.
-"Jog at least 2 miles per day."
Cardiac effects of hyperthyroidism include
-palpitations.
-decreased pulse pressure.
Cardiac effects may include sinus tachycardia, increased pulse
-decreased systolic blood pressure.
pressure, and palpitations. Systolic blood pressure is elevated.
-bradycardia.
-palpitations.
A client is scheduled for a diagnostic test to measure blood hor-
-The functioning of endocrine glands
mone levels. The nurse expects that this test will determine which
Measuring blood hormone levels helps determine the function-
of the following?
ing of endocrine glands. A radioimmunoassay determines the
concentration of a substance in plasma. The measurement of
-The concentration of a substance in plasma
blood hormone levels will not reveal a client's blood sugar level.
-Details about the size of the organ and its location
Radiographs of the chest or abdomen determine the size of the
-The functioning of endocrine glands
organ and its location.
-The client's blood sugar level
The nurse is reviewing a client's history which reveals that the
client has had an oversecretion of growth hormone (GH) that
-Gigantism
occurred before puberty. The nurse interprets this as which of the
When oversecretion of GH occurs before puberty, gigantism re-
following?
sults. Dwarfism occurs when secretion of GH is insufficient dur-
ing childhood. Oversecretion of GH during adulthood results in
-Gigantism
acromegaly. An absence of pituitary hormonal activity causes
-Dwarfism
Simmonds' disease.
-Acromegaly
-Simmonds' disease
Trousseau's sign is elicited by which of the following? -Carpopedal spasm is induced by occluding the blood flow to the
arm for 3 minutes with the use of a blood pressure cuff.
-Carpopedal spasm is induced by occluding the blood flow to the A positive Trousseau's sign is suggestive of latent tetany. A positive
arm for 3 minutes with the use of a blood pressure cuff. Chvostek's sign is demonstrated when a sharp tapping over the
-A sharp tapping over the facial nerve just in front of the parotid facial nerve just in front of the parotid gland and anterior to the ear
gland and anterior to the ear causes spasm or twitching of the causes spasm or twitching of the mouth, nose, and eye. A positive
mouth, nose, and eye. Allen's test is demonstrated by the palm remaining blanched with
-After making a clenched fist, the palm remains blanched when the radial artery occluded. The radial artery should not be used
pressure is placed over the radial artery. for an arterial puncture. A positive Homans' sign is demonstrated
-The patient complains of pain in the calf when his foot is dorsi- when the patient complains of pain in the calf when his foot is
flexed. dorsiflexed.
A client has been diagnosed with myxedema from long-standing
-Hypothermia
hypothyroidism. What clinical manifestations of this disorder does
-Hypotension
the nurse recognize are progressing to myxedema coma? Select
-Hypoventilation
all that apply.
Severe hypothyroidism is called myxedema. Advanced, untreat-
ed myxedema can progress to myxedemic coma. Signs of
-Hypothermia
this life-threatening event are hypothermia, hypotension, and
-Hypertension
hypoventilation. Hypertension and hyperventilation indicate in-
-Hypotension
creased metabolic responses, which are the opposite of what the
-Hypoventilation
client would be experiencing.
-Hyperventilation

A client with severe hypoparathyroidism is experiencing tetany.


-Calcium gluconate
What medication, prescribed by the physician for emergency use,
Tetany and severe hypoparathyroidismare treated immediately by
will the nurse administer to correct the deficit?
the administration of an IV calcium salt, such as calcium glu-
conate. The other medications are not effective for the treatment
-Sodium bicarbonate
of calcium deficit.
-Fludrocortisone

1/9
PrepU Chapter 52: Endocrine (Exam 1)
Study online at https://quizlet.com/_618nao
-Calcium gluconate
-Methylprednisolone
Cardiac effects of hyperthyroidism include
-palpitations.
-decreased pulse pressure.
Cardiac effects may include sinus tachycardia, increased pulse
-decreased systolic BP.
pressure, and palpitations. Systolic BP is elevated.
-bradycardia.
-palpitations.
The nurse practitioner who assesses a patient with hyperthy-
roidism would expect the patient to report which of the following
conditions?
-Weight loss
Weight loss is consistent with a diagnosis of hyperthyroidism. The
-Fatigue
other conditions are found in hypothyroidism.
-Dyspnea
-Weight loss
-Hair loss
The nurse assesses a patient who has been diagnosed with
Addison's disease. Which of the following is a diagnostic sign of
this disease? -Potassium of 6.0 mEq/L
Addison's disease is characterized by hypotension, low blood
-Potassium of 6.0 mEq/L glucose, low serum sodium, and high serum potassium levels. The
-Sodium of 140 mEq/L normal serum potassium level is 3.5 to 5 mEq/L.
-Glucose of 100 mg/dL
-A blood pressure reading of 135/90 mm Hg
Which feature(s) indicates a carpopedal spasm in a client with
hypoparathyroidism? -Hand flexing inward
Carpopedal spasm is evidenced by the hand flexing inward. Car-
-Hand flexing inward diac dysrhythmia is a symptom of hyperparathyroidism. Moon face
-Cardiac dysrhythmia and buffalo hump are the symptoms of Cushing syndrome. A
-Moon face and buffalo hump bulging forehead is a symptom of acromegaly.
-Bulging forehead
-Endocrine disorders can be inherited.
Some endocrine disorders are inherited or have a tendency to run
The nurse obtains a complete family history of a client with a
in families. Therefore, it is essential to take a complete family histo-
suspected endocrine disorder based on which rationale?
ry. A complete blood count and chemistry profile are performed to
determine the client's general status and to rule out disorders. Ob-
-An allergy to iodine is inherited.
taining information about an allergy to iodine is important because
-Endocrine disorders can be inherited.
diagnostic testing may involve the use of contrast dyes. However,
-It helps determine the client's general status.
an allergy to iodine is not related to endocrine disorders. Diet and
-Diet and drug histories are related to the family history.
drug histories, although important information, are not associated
with the family history.
A client presents with a huge lower jaw, bulging forehead, large -hyperpituitarism
hands and feet, and frequent headaches. What could be causing Acromegaly (hyperpituitarism) is a condition in which growth hor-
this client's symptoms? mone is oversecreted after the epiphyses of the long bones have
sealed. A client with acromegaly has coarse features, a huge lower
-hyperpituitarism jaw, thick lips, a thickened tongue, a bulging forehead, a bulbous
-hypopituitarism nose, and large hands and feet. When the overgrowth is from a
-panhypopituitarism tumor, headaches caused by pressure on the sella turcica are
-panhyperpituitarism common.
A nurse is caring for a client with hypoparathyroidism. During
assessment, the nurse elicits a positive Trousseau's sign. What
does the nurse observe to verify this finding?
-hand flexing inward
The nurse observes the client for spasm of the hand (carpopedal
-hand flexing inward
spasm), which is evidenced by the hand flexing inward.
-cardiac dysrhythmia
-moon face and buffalo hump
-bulging forehead

A client has been experiencing a decrease in serum calcium. After


diagnostics, the physician proposes the calcium level fluctuation

2/9
PrepU Chapter 52: Endocrine (Exam 1)
Study online at https://quizlet.com/_618nao
is due to altered parathyroid function. What is the typical number
of parathyroid glands? -four
The parathyroid glands are four (some people have more than
-four four) small, bean-shaped bodies, each surrounded by a capsule
-three of connective tissue and embedded within the lateral lobes of the
-two thyroid.
-one
-computed tomography scan
Which diagnostic test is done to determine suspected pituitary
A computed tomography or magnetic resonance imaging scan is
tumor?
done to detect a suspected pituitary tumor. Radiographs of the
chest or abdomen are taken to detect tumors. Radiographs also
-computed tomography scan
determine the size of the organ and their location. Measuring blood
-measurement of blood hormone levels
hormone levels helps determine the functioning of endocrine
-radioimmunoassay
glands. A radioimmunoassay determines the concentration of a
-radiographs of the abdomen
substance in plasma.
A client with acromegaly has been given the option of a surgical
approach or a medical approach. The client decides to have a -Hypophysectomy
surgical procedure to remove the pituitary gland. What does the The treatment of choice is surgical removal of the pituitary gland
nurse understand this surgical procedure is called? (transsphenoidal hypophysectomy) through a nasal approach.
The surgeon may substitute an endoscopic technique using mi-
-Hypophysectomy crosurgical instruments to reduce surgical trauma. A hysteroscopy
-Hysteroscopy is a gynecologic procedure. The thyroid gland is not involved for a
-Thyroidectomy surgical procedure. Ablation is not a removal of the pituitary gland.
-Ablation
A client with a traumatic brain injury is producing an abnormally
-Deficient production of vasopressin
large volume of dilute urine. Which alteration to a hormone se-
The most common disorder related to posterior lobe dysfunction is
creted by the posterior pituitary would the nurse expect to find?
diabetes insipidus, a condition in which abnormally large volumes
of dilute urine are excreted as a result of deficient production
-Deficient production of vasopressin
of vasopressin. Diabetes insipidus may occur following surgical
-Increased antidiuretic hormone
treatment of a brain tumor, secondary to nonsurgical brain tumors,
-Increased oxytocin
and traumatic brain injury.
-A deficient amount of somatostatin
During an assessment of a client's functional health pattern, which
question by the nurse directly addresses the client's thyroid func-
tion? -"Do you experience fatigue even if you have slept a long time?"
With the diagnosis of hypothyroidism, extreme fatigue makes it
-"Do you have to get up at night to empty your bladder?" difficult for the person to complete a full day's work or participate
-"Have you experienced any headaches or sinus problems?" in usual activities.
-"Do you experience fatigue even if you have slept a long time?"
-"Can you describe the amount of stress in your life?"
A health care provider suspects that a thyroid nodule may be
malignant. The nurse knows to prepare information for the patient
-Fine-needle biopsy of the thyroid gland
based on the usual test that will be ordered to establish a diag-
Fine needle biopsy of the thyroid gland is often used to establish
nosis. What is that test?
the diagnosis of thyroid cancer. The purpose of the biopsy is to
differentiate cancerous thyroid nodules from noncancerous nod-
-Serum immunoassay for TSH
ules and to stage the cancer if detected. The procedure is safe
-Fine-needle biopsy of the thyroid gland
and usually requires only a local anesthetic.
-Free T4 analysis
-Ultrasound of the thyroid gland

A client with a history of chronic hyperparathyroidism admits to


-Related to bone demineralization resulting in pathologic fractures
being noncompliant. Based on initial assessment findings, the
Poorly controlled hyperparathyroidism may cause an elevated
nurse formulates the nursing diagnosis of Risk for injury. To com-
serum calcium level. This increase, in turn, may diminish calcium
plete the nursing diagnosis statement for this client, which "relat-
stores in the bone, causing bone demineralization and setting the
ed-to" phrase should the nurse add?
stage for pathologic fractures and a risk for injury. Hyperparathy-
roidism doesn't accelerate the metabolic rate. A decreased thyroid
-Related to bone demineralization resulting in pathologic frac-
hormone level, not an increased parathyroid hormone level, may
tures
cause edema and dry skin secondary to fluid infiltration into the
-Related to exhaustion secondary to an accelerated metabolic
interstitial spaces. Hyperparathyroidism causes hypercalcemia,
rate
not hypocalcemia; therefore, it isn't associated with tetany.
-Related to edema and dry skin secondary to fluid infiltration into
3/9
PrepU Chapter 52: Endocrine (Exam 1)
Study online at https://quizlet.com/_618nao
the interstitial spaces
-Related to tetany secondary to a decreased serum calcium level
-decreased body temperature and cold intolerance.
A client is being evaluated for hypothyroidism. During assessment,
Hypothyroidism markedly decreases the metabolic rate, causing
the nurse should stay alert for:
a reduced body temperature and cold intolerance. Other signs
and symptoms include dyspnea, hypoventilation, bradycardia, hy-
-exophthalmos and conjunctival redness.
potension, anorexia, constipation, decreased intellectual function,
-flushed, warm, moist skin.
and depression. Exophthalmos; conjunctival redness; flushed,
-systolic murmur at the left sternal border.
warm, moist skin; and a systolic murmur at the left sternal border
-decreased body temperature and cold intolerance.
are typical findings in a client with hyperthyroidism.
-Serum potassium level of 6.8 mEq/L
A serum potassium level of 6.8 mEq/L indicates hyperkalemia,
A nurse is caring for a client in acute addisonian crisis. Which test which can occur in adrenal insufficiency as a result of reduced al-
result does the nurse expect to see? dosterone secretion. A BUN level of 2.3 mg/dl is lower than normal.
A client in addisonian crisis is likely to have an increased BUN level
-Serum potassium level of 6.8 mEq/L because the glomerular filtration rate is reduced. A serum sodium
-Blood urea nitrogen (BUN) level of 2.3 mg/dl level of 156 mEq/L indicates hypernatremia. Hyponatremia is more
-Serum sodium level of 156 mEq/L likely in this client because of reduced aldosterone secretion. A
-Serum glucose level of 236 mg/dl serum glucose level of 236 mg/dl indicates hyperglycemia. This
client is likely to have hypoglycemia caused by reduced cortisol
secretion, which impairs glyconeogenesis.
A client is receiving long-term treatment with high-dose corticos-
teroids. Which of the following would the nurse expect the client -Moon face
to exhibit? Clients who are receiving long-term high-dose corticosteroid ther-
apy often develop a cushingoid appearance, manifested by facial
-Weight loss fullness and the characteristic moon face. They also may exhibit
-Pale thick skin weight gain, peripheral edema, and hypertension due to sodium
-Moon face and water retention. The skin is usually thin, and ruddy.
-Hypotension
A nurse is reviewing the laboratory order for a client suspected
of having an endocrine disorder. The lab slip includes obtaining
cortisol levels. What is being tested? -adrenal function
The adrenal cortex manufactures and secretes glucocorticoids,
-adrenal function such as cortisol, which affect body metabolism, suppress inflam-
-thyroid function mation, and help the body withstand stress.
-thymus function
-parathyroid function
A client sustained a head injury when falling off of a ladder. While
in the hospital, the client begins voiding large amounts of clear -Diabetes insipidus (DI)
urine and states he is very thirsty. The client states that he feels Urine output may be as high as 20 L/24 hours. Urine is dilute, with
weak, and he has had an 8-lb weight loss since admission. What a specific gravity of 1.002 or less. Limiting fluid intake does not
should the client be tested for? control urine exertion. Thirst is excessive and constant. Activities
are limited by the frequent need to drink and void. Weakness, de-
-Diabetes insipidus (DI) hydration, and weight loss develop. SIADH will have the opposite
-Syndrome of inappropriate antidiuretic hormone secretion clinical manifestations. The client's symptoms are related to the
(SIADH) trauma and not a pituitary tumor. The thyroid gland does not exhibit
-Pituitary tumor these symptoms.
-Hypothyroidism
A client has a decreased level of thyroid hormone being excreted.
What will the feedback loop do to maintain the level of thyroid
hormone required to maintain homeostatic stability?
-Stimulate more hormones using the negative feedback system
Feedback can be either negative or positive. Most hormones are
-Stimulate more hormones using the negative feedback system
secreted in response to negative feedback; a decrease in levels
-Stimulate more hormones using the positive feedback system
stimulates the releasing gland.
-Produce a new hormone to try and regulate the thyroid function
-The feedback loop will be unable to perform in response to low
levels of thyroid hormone.
-Cushing syndrome
Which disorder is characterized by a group of symptoms produced
The client with Cushing syndrome demonstrates truncal obesity,
by an excess of free circulating cortisol from the adrenal cortex?
moon face, acne, abdominal striae, and hypertension. Regardless
4/9
PrepU Chapter 52: Endocrine (Exam 1)
Study online at https://quizlet.com/_618nao
of the cause, the normal feedback mechanisms that control the
function of the adrenal cortex become ineffective, and the usual
-Cushing syndrome
diurnal pattern of cortisol is lost. The signs and symptoms of
-Addison disease
Cushing syndrome are primarily a result of the oversecretion of
-Graves disease
glucocorticoids and androgens, although mineralocorticoid secre-
-Hashimoto disease
tion also may be affected.
The nurse is caring for a patient with hyperthyroidism who sud- -Temperature of 102ºF
denly develops symptoms related to thyroid storm. What symp- Thyroid storm is characterized by the following: 1) high fever
toms does the nurse recognize that are indicative of this emer- (hyperpyrexia), >38.5°C (>101.3°F); 2) extreme tachycardia (>130
gency? bpm); 3) exaggerated symptoms of hyperthyroidism with distur-
bances of a major system—for example, gastrointestinal (weight
-Heart rate of 62 loss, diarrhea, abdominal pain) or cardiovascular (edema, chest
-Blood pressure 90/58 mm Hg pain, dyspnea, palpitations); and 4) altered neurologic or mental
-Oxygen saturation of 96% state, which frequently appears as delirium psychosis, somno-
-Temperature of 102ºF lence, or coma.
A nurse is assigned to care for a patient with increased parathor-
mone secretion. Which of the following serum levels should the
nurse monitor for this patient?
-Calcium
Increased secretion of parathormone results in bone resorption.
-Glucose
Calcium is released into the blood, increasing serum levels.
-Sodium
-Calcium
-Potassium
-Decreased cardiac output
An acute addisonian crisis is a life-threatening event, caused by
A nurse is planning care for a client in acute addisonian crisis. deficiencies of cortisol and aldosterone. Glucocorticoid insuffi-
Which nursing diagnosis should receive the highest priority? ciency causes a decrease in cardiac output and vascular tone,
leading to hypovolemia. The client becomes tachycardic and hy-
-Risk for infection potensive and may develop shock and circulatory collapse. The
-Decreased cardiac output client with Addison's disease is at risk for infection; however,
-Impaired physical mobility reducing infection isn't a priority during an addisonian crisis. Im-
-Imbalanced nutrition: Less than body requirements paired physical mobility and Imbalanced nutrition: Less than body
requirements are appropriate nursing diagnoses for the client with
Addison's disease, but they aren't priorities in a crisis.
Trousseau sign is elicited -by occluding the blood flow to the arm for 3 minutes with the use
of a blood pressure cuff.
-by occluding the blood flow to the arm for 3 minutes with the use A positive Trousseau sign is suggestive of latent tetany. A positive
of a blood pressure cuff. Chvostek sign is demonstrated when a sharp tapping over the
-by tapping sharply over the facial nerve just in front of the parotid facial nerve just in front of the parotid gland and anterior to the ear
gland and anterior to the ear, causing spasm or twitching of the causes the mouth, nose, and eye to spasm or twitch. The palm
mouth, nose, and eye. remaining blanched when the radial artery is occluded demon-
-after making a clenched fist and opening the hand; the palm strates a positive Allen test. The radial artery should not be used
remains blanched when pressure is placed over the radial artery. for an arterial puncture. A positive Homans sign is demonstrated
-when the foot is dorsiflexed and there is pain in the calf. when the client reports pain in the calf when the foot is dorsiflexed.
The most common cause of hypothyroidism is which of the follow-
-Autoimmune thyroiditis
ing?
The most common cause of hypothyroidism is autoimmune thy-
roiditis (Hashimoto's disease), in which the immune system at-
-Autoimmune thyroiditis
tacks the thyroid gland. Hypothyroidism can occur in patient with
-Radioiodine therapy
previous hyperthyroidism that has been treated with radioiodine,
-Antithyroid medications
antithyroid medication treatment, or thyroidectomy.
-Thyroidectomy
Accidental removal of one or both parathyroid glands can occur
during a thyroidectomy. Which of the following is used to treat -Calcium gluconate
tetany? Sometimes in thyroid surgery, the parathyroid glands are removed,
producing a disturbance in calcium metabolism. Tetany is usu-
-Calcium gluconate ally treated with IV calcium gluconate. Synthroid is used in the
-Synthroid treatment of hypothyroidism. PTU and Tapazole are used in the
-Propylthiouracil (PTU) treatment of hyperthyroidism.
-Tapazole

5/9
PrepU Chapter 52: Endocrine (Exam 1)
Study online at https://quizlet.com/_618nao
Antithyroid medications are contraindicated in late pregnancy due
to the fact that which of the following may occur? Select all that
apply.
-Fetal hypothyroidism
Antithyroid medications are contraindicated in late pregnancy be-
-Fetal hypothyroidism
cause the fetus may develop fetal hypothyroidism, fetal bradycar-
-Fetal bradycardia
dia, goiter, and cretinism.
-Goiter
-Cretinism
-Fetal tachycardia
Which assessment would a nurse perform on a client with Cush-
ing's syndrome who is at high risk of developing a peptic ulcer?
-Observe stool color.
-Observe stool color. The nurse should observe the color of each stool and test the stool
-Monitor bowel patterns. for occult blood.
-Monitor vital signs every 4 hours.
-Observe urine output.
-Provide a high-sodium diet.
A nurse is caring for a client with Cushing's syndrome. Which
Limiting sodium reduces the potential for fluid retention. Fluid
would the nurse not include in this client's plan of care?
retention is manifested by swelling in dependent areas, pitting
when pressure is applied to the skin over a bone by tight-fitting
-Provide a high-sodium diet.
shoes or rings, the appearance of lines in the skin from stockings
-Examine extremities for pitting edema.
and seams in the shoes or areas where they lace. Hypertension
-Report systolic BP that exceeds 139 mm Hg or diastolic BP that
is defined as a consistently elevated BP above 139/89 mm Hg.
exceeds 89 mm Hg.
One factor that contributes to hypertension is excess circulatory
-Administer prescribed diuretics.
volume. Diuretics promote the excretion of sodium and water.
The nurse is reviewing a client's history which reveals that the
client has had an over secretion of growth hormone (GH) that
-Gigantism
occurred before puberty. The nurse interprets this as which of the
When over secretion of GH occurs before puberty, gigantism
following?
results. Dwarfism occurs when secretion of GH is insufficient
during childhood. Oversecretion of GH during adulthood results
-Gigantism
in acromegaly. An absence of pituitary hormonal activity causes
-Dwarfism
Simmonds' disease.
-Acromegaly
-Simmonds' disease
What interventions can the nurse encourage the client with dia-
betes insipidus to do in order to control thirst and compensate for -Consume adequate amounts of fluid.
urine loss? The nurse teaches the client to consume sufficient fluid to control
thirst and to compensate for urine loss. The client will not be
-Come to the clinic for IV fluid therapy daily. required to come in daily for IV fluid therapy. The client should not
-Limit the fluid intake at night. limit fluid intake at night if thirst is present. Weighing daily will not
-Consume adequate amounts of fluid. control thirst or compensate for urine loss.
-Weigh daily.
-Maintaining room temperature in the low-normal range
Graves' disease causes signs and symptoms of hypermetabolism,
For a client with Graves' disease, which nursing intervention pro-
such as heat intolerance, diaphoresis, excessive thirst and ap-
motes comfort?
petite, and weight loss. To reduce heat intolerance and diaphore-
sis, the nurse should keep the client's room temperature in the
-Restricting intake of oral fluids
low-normal range. To replace fluids lost via diaphoresis, the nurse
-Placing extra blankets on the client's bed
should encourage, not restrict, intake of oral fluids. Placing extra
-Limiting intake of high-carbohydrate foods
blankets on the bed of a client with heat intolerance would cause
-Maintaining room temperature in the low-normal range
discomfort. To provide needed energy and calories, the nurse
should encourage the client to eat high-carbohydrate foods.

A nurse is assessing a client after a thyroidectomy. The assess- -Tetany


ment reveals muscle twitching and tingling, along with numbness Tetany may result if the parathyroid glands are excised or dam-
in the fingers, toes, and mouth area. The nurse should suspect aged during thyroid surgery. Hemorrhage is a potential compli-
which complication? cation after thyroid surgery but is characterized by tachycardia,
hypotension, frequent swallowing, feelings of fullness at the in-
-Tetany cision site, choking, and bleeding. Thyroid storm is another term
-Hemorrhage for severe hyperthyroidism — not a complication of thyroidectomy.

6/9
PrepU Chapter 52: Endocrine (Exam 1)
Study online at https://quizlet.com/_618nao
-Thyroid storm Laryngeal nerve damage may occur postoperatively, but its signs
-Laryngeal nerve damage include a hoarse voice and, possibly, acute airway obstruction.
A client receiving thyroid replacement therapy develops influenza
-Myxedema coma
and forgets to take her thyroid replacement medicine. The nurse
Myxedema coma, severe hypothyroidism, is a life-threatening
understands that skipping this medication puts the client at risk
condition that may develop if thyroid replacement medication isn't
for developing which life-threatening complication?
taken. Exophthalmos (protrusion of the eyeballs) is seen with
hyperthyroidism. Although thyroid storm is life-threatening, it's
-Exophthalmos
caused by severe hyperthyroidism. Tibial myxedema (peripheral
-Thyroid storm
mucinous edema involving the lower leg) is associated with hy-
-Myxedema coma
pothyroidism but isn't life-threatening.
-Tibial myxedema
During physical examination of a client with a suspected en-
-Detecting evidence of hormone hypersecretion.
docrine disorder, the nurse assesses the body structures. The
The evaluation of body structures helps the nurse detect evidence
nurse gathers this data based on the understanding that it is an
of hypersecretion or hyposecretion of hormones. This helps in
important aid in which of the following?
the assessment of findings that are unique to specific endocrine
glands. Radiographs of the chest or abdomen are taken to detect
-Detecting evidence of hormone hypersecretion.
tumors. Radiographs also determine the size of the organ and its
-Detecting information about possible tumor growth.
location. Antidiuretic hormone (ADH) levels determine the pres-
-Determining the presence or absence of testosterone levels.
ence or absence of ADH and testosterone levels.
-Determining the size of the organs and location.
When describing the difference between endocrine and exocrine -The secretions are released directly into the blood stream.
glands, which of the following would the instructor include as The endocrine glands secrete hormones, chemicals that acceler-
characteristic of endocrine glands? ate or slow physiologic processes, directly into the bloodstream.
This characteristic distinguishes endocrine glands from exocrine
-The secretions are released directly into the blood stream. glands, which release secretions into a duct. Hormones circulate
-The glands contain ducts that produce the hormones. in the blood until they reach receptors in target cells or other
-The secreted hormones act like target cells. endocrine glands. They play a vital role in regulating homeostatic
-The glands play a minor role in maintaining homeostasis. processes.
-prolactin
A nurse educator is teaching a chapter on "The Function of the Hypothalamic dopamine inhibits the release of prolactin from the
Endocrine System." Which hormone would not be included as one anterior pituitary gland. Corticotropin-releasing hormone (CRH)
of the six hypothalamic hormones? causes the anterior pituitary gland to secrete adrenocorticotropic
hormone (ACTH). Thyrotropin-releasing hormone (TRH) stimu-
-prolactin lates the release of thyroid-stimulating hormone (TSH) from the
-corticotropin-releasing hormone anterior pituitary gland. Gonadotropin-releasing hormone (GnRH)
-thyrotropin-releasing hormone triggers sexual development at the onset of puberty and continues
-gonadotropin-releasing hormone to cause the anterior pituitary gland to secrete luteinizing hormone
(LH) and follicle-stimulating hormone (FSH).
A patient is suspected of having a pheochromocytoma and is
having diagnostic tests done in the hospital. What symptoms does -Blood pressure varying between 120/86 and 240/130 mm Hg
the nurse recognize as most significant for a patient with this Hypertension associated with pheochromocytoma may be inter-
disorder? mittent or persistent. Blood pressures exceeding 250/150 mm
Hg have been recorded. Such blood pressure elevations are life
-Blood pressure varying between 120/86 and 240/130 mm Hg threatening and can cause severe complications, such as cardiac
-Heart rate of 56-64 bpm dysrhythmias, dissecting aneurysm, stroke, and acute kidney fail-
-Shivering ure.
-Complaints of nausea
A patient taking corticosteroids for exacerbation of Crohn's dis-
ease comes to the clinic and informs the nurse that he wants -The moon face and acne will resolve when the medication is
to stop taking them because of the increase in acne and moon tapered off.
face. What can the nurse educate the patient regarding these Cushing syndrome is commonly caused by the use of corticos-
symptoms? teroid medications and is infrequently the result of excessive
corticosteroid production secondary to hyperplasia of the adrenal
-The symptoms are permanent side effects of the corticosteroid cortex. The patient develops a "moon-faced" appearance and may
therapy. experience increased oiliness of the skin and acne. If Cushing
-The moon face and acne will resolve when the medication is syndrome is a result of the administration of corticosteroids, an
tapered off. attempt is made to reduce or taper the medication to the minimum
-Those symptoms are not related to the corticosteroid therapy. dosage needed to treat the underlying disease process (e.g.,
-The dose of the medication must be too high and should be autoimmune or allergic disease, rejection of a transplanted organ).
lowered.

7/9
PrepU Chapter 52: Endocrine (Exam 1)
Study online at https://quizlet.com/_618nao
The nurse knows to assess a patient with hyperthyroidism for the
primary indicator of:
-Intolerance to heat
With hypothyroidism, the individual is sensitive to cold because
-Fatigue
the core body temperature is usually below 98.6°F. Intolerance to
-Weight gain
heat is seen with hyperthyroidism.
-Constipation
-Intolerance to heat
Patients with hyperthyroidism are characteristically:
-Sensitive to heat
-Apathetic and anorexic Those with hyperthyroidism tolerate heat poorly and may perspire
-Calm unusually freely. Their condition is characterized by symptoms of
-Emotionally stable nervousness, hyperexcitability, irritability, and apprehension.
-Sensitive to heat
-An irregular apical pulse
A nurse is assessing a client with Cushing's syndrome. Which Because Cushing's syndrome causes aldosterone overproduc-
observation should the nurse report to the physician immediately? tion, which increases urinary potassium loss, the disorder may
lead to hypokalemia. Therefore, the nurse should immediately
-Pitting edema of the legs report signs and symptoms of hypokalemia, such as an irregu-
-An irregular apical pulse lar apical pulse, to the physician. Edema is an expected finding
-Dry mucous membranes because aldosterone overproduction causes sodium and fluid
-Frequent urination retention. Dry mucous membranes and frequent urination signal
dehydration, which isn't associated with Cushing's syndrome.
-Fluid intake is less than 2,500 ml/day.
Which outcome indicates that treatment of a client with diabetes
Diabetes insipidus is characterized by polyuria (up to 8 L/day),
insipidus has been effective?
constant thirst, and an unusually high oral intake of fluids. Treat-
ment with the appropriate drug should decrease both oral fluid
-Fluid intake is less than 2,500 ml/day.
intake and urine output. A urine output of 200 ml/hour indicates
-Urine output measures more than 200 ml/hour.
continuing polyuria. A blood pressure of 90/50 mm Hg and a heart
-Blood pressure is 90/50 mm Hg.
rate of 126 beats/minute indicate compensation for the continued
-Heart rate is 126 beats/minute.
fluid deficit, suggesting that treatment hasn't been effective.
A nurse is caring for a client who had a thyroidectomy and is at
risk for hypocalcemia. What should the nurse do?
-Observe for muscle twitching and numbness or tingling of the lips,
-Monitor laboratory values daily for elevated thyroid-stimulating fingers, and toes.
hormone. Muscle twitching and numbness or tingling of the lips, fingers,
-Observe for swelling of the neck, tracheal deviation, and severe and toes are signs of hyperirritability of the nervous system due
pain. to hypocalcemia. The other options describe complications for
-Evaluate the quality of the client's voice postoperatively, noting which the nurse should also be observing; however, tetany and
any drastic changes. neurologic alterations are primary indications of hypocalcemia.
-Observe for muscle twitching and numbness or tingling of the lips,
fingers, and toes.
A 35-year-old female client who complains of weight gain, facial
hair, absent menstruation, frequent bruising, and acne is diag- -a corticotropin-secreting pituitary adenoma.
nosed with Cushing's syndrome. Cushing's syndrome is most A corticotropin-secreting pituitary adenoma is the most common
likely caused by: cause of Cushing's syndrome in women ages 20 to 40. Ectopic
corticotropin-secreting tumors are more common in older men and
-an ectopic corticotropin-secreting tumor. are commonly associated with weight loss. Adrenal carcinoma
-adrenal carcinoma. isn't usually accompanied by hirsutism. A female with an inborn
-a corticotropin-secreting pituitary adenoma. error of metabolism wouldn't be menstruating.
-an inborn error of metabolism.
Which hormone is secreted by the posterior pituitary?
-Vasopressin
Vasopressin causes smooth muscle, particularly blood vessels, to
-Vasopressin
contract. Calcitonin is secreted by the parafollicular cells of the
-Calcitonin
thyroid gland. Corticosteroids are secreted by the adrenal cortex.
-Corticosteroids
Somatostatin is released by the anterior lobe of the pituitary.
-Somatostatin
-Pineal gland, melatonin
A client visits the clinic to seek treatment for disturbed sleep
The pineal gland secretes melatonin, which aids in regulating
cycles and depressed mood. Which glands and hormones help
sleep cycles and mood. Melatonin plays a vital role in hypo-
to regulate sleep cycles and mood?
thalamicpituitary interaction. The thymus gland secretes thymosin
8/9
PrepU Chapter 52: Endocrine (Exam 1)
Study online at https://quizlet.com/_618nao
and thymopoietin, which aid in developing T lymphocytes. The
-Thymus gland, thymosin parathyroid glands secrete parathormone, which increases the
-Parathyroid glands, parathormone levels of calcium and phosphorus in the blood. The adrenal cortex
-Pineal gland, melatonin secretes corticosteroids hormones, which influence many organs
-Adrenal cortex, corticosteroids and structures of the body.
An instructor is preparing a teaching plan for a class on the various
pituitary hormones. Which hormone would the instructor include
as being released by the posterior pituitary gland? -Oxytocin
The posterior pituitary gland released oxytocin and antidiuretic
-Somatotropin hormone. Somatotropin, prolactin, and adrenocorticotropic hor-
-Prolactin mone are released by the anterior pituitary gland.
-Oxytocin
-Adrenocorticotropic hormone
Although not designated as endocrine glands, several organs
within the body secrete hormones as part of their normal function.
Which organ secretes hormones involved in increasing blood -kidneys
pressure and volume and maturation of red blood cells? The kidneys release renin, a hormone that initiates the production
of angiotensin and aldosterone to increase blood pressure and
-kidneys blood volume. The kidneys also secrete erythropoietin, a sub-
-cardiac atria stance that promotes the maturation of red blood cells.
-brain
-liver
A client is suspected to have a pituitary tumor due to signs of
-Magnetic resonance imaging (MRI)
diabetes insipidus. What initial test does the nurse help to prepare
A computed tomography (CT) or magnetic resonance imaging
the client for?
(MRI) scan is performed to detect a suspected pituitary tumor
or to identify calcifications or tumors of the parathyroid glands. A
-Magnetic resonance imaging (MRI)
radioactive iodine uptake test would be useful for a thyroid tumor.
-Radioactive iodine uptake test
Radioimmunoassay determines the concentration of a substance
-Radioimmunoassay
in plasma.
-A nuclear scan
-A decrease in urine output
Desmopressin (DDAVP), a synthetic vasopressin without the vas-
A patient is ordered desmopressin (DDAVP) for the treatment cular effects of natural ADH, is particularly valuable because it
of diabetes insipidus. What therapeutic response does the nurse has a longer duration of action and fewer adverse effects than
anticipate the patient will experience? other preparations previously used to treat the disease. DDAVP
and lypressin (Diapid) reduce urine output to 2 to 3 L/24 hours.
-A decrease in blood pressure It is administered intranasally; the patient sprays the solution into
-A decrease in blood glucose levels the nose through a flexible calibrated plastic tube. One or two
-A decrease in urine output administrations daily (i.e., every 12 to 24 hours) usually control
-A decrease in appetite the symptoms (Papadakis, McPhee, & Rabow, 2013). Vasopressin
causes vasoconstriction; thus, it must be used cautiously in pa-
tients with coronary artery disease.

9/9
Unit of 3: Endocrine & Metabolic Disorders
Study online at https://quizlet.com/_8igkej
polyuria
When obtaining the nursing history of a client who has diabetes
mellitus, the nurse should assess the client for which of the fol- In early renal insufficiency, the kidneys lose the ability to concen-
lowing early symptom of renal insufficiency? trate urine, resulting in polyuria. Oliguria occurs later. Dysuria and
hematuria are not associated with renal insufficiency.
"You must avoid hyperextending your neck after surgery."

To prevent undue pressure on the surgical incision after subtotal


During preoperative teaching for a client who will undergo subtotal thyroidectomy, the nurse should advise the client to avoid hy-
thyroidectomy, the nurse should include which statement? perextending the neck. The client may elevate the head of the
bed as desired and should perform deep breathing and coughing
to help prevent pneumonia. Subtotal thyroidectomy doesn't affect
swallowing.
colchicine

A disease characterized by joint inflammation (especially in the


great toe), gout is caused by urate crystal deposits in the joints.
The physician orders colchicine to reduce these deposits and
A nurse should expect to administer which medication to a client
thus ease joint inflammation. Although aspirin reduces joint in-
with gout?
flammation and pain in clients with osteoarthritis and rheumatoid
arthritis, it isn't indicated for gout because it has no effect on
urate crystal formation. Furosemide, a diuretic, doesn't relieve
gout. Calcium gluconate reverses a negative calcium balance and
relieves muscle cramps; it doesn't treat gout.
"It's important to dry my feet carefully after my bath."

It is important to dry the feet carefully after a bath to prevent a fun-


A client with diabetes is explaining to the nurse how to care
gal infection. Clients with diabetes should seek medical attention
for the feet at home. Which statement indicates that the client
when they injure their toes or feet to prevent complications. Iodine
understands proper foot care?
is highly toxic to the tissues. Clients with diabetes should inspect
their feet daily and should wear shoes that support their feet while
in the house.
obtaining adequate food intake

The priority goal for the client with diabetes mellitus who is ex-
periencing vomiting with influenza is to obtain adequate nutrition.
The diabetic client should eat small, frequent meals of 50 g of
carbohydrate or food equal to 200 cal every 3 to 4 hours. If the
Which goal is a priority for the diabetic client who is taking insulin client cannot eat the carbohydrates or take fluids, the health care
and has nausea and vomiting from a viral illness or influenza? provider (HCP) should be called, or the client should go to the
emergency department. The diabetic client is in danger of compli-
cations with dehydration, electrolyte imbalance, and ketoacidosis.
Increasing the client's health management skills is important to
lifestyle behaviors, but it is not a priority during this acute illness of
influenza. Pain relief may be a need for this client, but it is not the
priority at this time; neither is increasing activity during the illness.
"It will be necessary to take the medication for the rest of your life."

Thyroid replacement is a lifelong maintenance therapy. The med-


A client newly diagnosed with hypothyroidism asks the nurse how
ication is usually given as one dose in the morning. It cannot be
long it will be necessary to take the prescribed synthroid. What
tapered or discontinued because the client needs thyroid supple-
should the nurse tell the client?
mentation to maintain health. The medication cannot be discon-
tinued after the TSH level is normal; the dose will be maintained
at the level that normalizes the TSH concentration.
concentration of urine

The major characteristic of diabetes insipidus is decreased tubular


reabsorption of water due to insufficient amounts of antidiuretic
hormone (ADH). Vasopressin is administered to the client with
diabetes insipidus because it has pressor and ADH activities.
1 / 27
Unit of 3: Endocrine & Metabolic Disorders
Study online at https://quizlet.com/_8igkej
Vasopressin works to increase the concentration of the urine by
increasing tubular reabsorption, thus preserving up to 90% water.
Vasopressin is administered to the client with diabetes insipidus
because it is a synthetic ADH. The administration of vasopressin
A client with diabetes insipidus is receiving vasopressin. Which
results in increased tubular reabsorption of water, and it is effective
sign indicates that the drug is having the intended effect?
for emergency treatment or daily maintenance of mild diabetes
insipidus. Vasopressin does not decrease blood pressure or affect
insulin production or glucose metabolism, nor is insulin production
a factor in diabetes insipidus.
sweating, tremors, and tachycardia
A client with status asthmaticus requires endotracheal intubation
and mechanical ventilation. Twenty-four hours after intubation, the Sweating, tremors, and tachycardia, thirst, and anxiety are early
client is started on the insulin infusion protocol. The nurse must signs of hypoglycemia. Dry skin, bradycardia, and somnolence
monitor the client's blood glucose levels hourly and watch for are signs and symptoms associated with hypothyroidism. Polyuria,
which early signs and symptoms associated with hypoglycemia? polydipsia, and polyphagia are signs and symptoms of diabetes
mellitus.
I'll stay here with you while the technician draws your blood."

The nurse should tell the client that they will stay with them as
the blood is drawn. This response provides the client with the
reassuring presence of the nurse and enhances the therapeutic
A client with hypothyroidism is afraid of needles and doesn't want
alliance, possibly providing a greater opportunity to educate the
to have their blood drawn. What should the nurse say to help
client. Although telling the client that blood won't need to be drawn
alleviate the client's concerns?
as often when thyroid levels are stable provides the client with
a rationale for needing blood work, it's more appropriate for the
nurse to stay with the client. Saying that the procedure will be over
quickly or that the physician has ordered the blood draw ignores
the client's stated fear.
chicken broth and juice

Electrolyte imbalances associated with Addison's disease include


A client with Addison's disease has fluid and electrolyte loss due to
hypoglycemia, hyponatremia, and hyperkalemia. Regular salted
inadequate fluid intake and to fluid loss secondary to inadequate
(not low salt) chicken or beef broth and fruit juices provide glucose
adrenal hormone secretion. As the client's oral intake increases,
and sodium to replenish these deficits. Diet soda does not contain
which fluids would be most appropriate?
sugar. Water could cause further sodium dilution. Coffee's diuretic
effect would aggravate the fluid deficit. Milk contains potassium
and sodium.
tachycardia
A client with hypothyroidism (myxedema) is receiving levothyrox-
Levothyroxine, a synthetic thyroid hormone, is given to a client
ine, 25 mcg P.O. daily. Which finding should the nurse recognize
with hypothyroidism to simulate the effects of thyroxine. Adverse
as an adverse reaction to the drug?
reactions to this agent include tachycardia. Dysuria, leg cramps,
and blurred vision aren't associated with levothyroxine.
glycosylated hemoglobin level.

Because some of the glucose in the bloodstream attaches to some


A client tells the nurse that they have been working hard for the of the hemoglobin and stays attached during the 120-day life span
past 3 months to control the client's type 2 diabetes with diet and of red blood cells, glycosylated hemoglobin levels provide infor-
exercise. To determine the effectiveness of the client's efforts, the mation about blood glucose levels during the previous 3 months.
nurse should check Fasting blood glucose and urine glucose levels give information
only about glucose levels at the point in time when they were
obtained. Serum fructosamine levels provide information about
blood glucose control over the past 2 to 3 weeks.

"I will increase fiber and fluids in my diet."


A client has been diagnosed with hypothyroidism. Which state-
Clients with hypothyroidism typically have constipation. A diet high
ment by the client would demonstrate appropriate teaching by the
in fiber and fluids can help prevent this. Group activities have
nurse?
nothing to do with the current issue. A nurse would not change
medical prescriptions by telling the client to stop taking the pre-

2 / 27
Unit of 3: Endocrine & Metabolic Disorders
Study online at https://quizlet.com/_8igkej
scribed aspirin. Increasing caloric consumption is not appropriate
with hypothyroidism.
the client with type 1 diabetes mellitus with acute visual changes
Which client will the community health nurse visit first?
The highest priority client is the one with acute vision problems.
The other clients need to be seen but are not emergent.
Write down the results, read back the results to the caller from the
laboratory, and receive confirmation from the caller.

To assure client safety, the nurse first writes the results on the
The nurse is receiving results of a blood glucose level from the chart, then reads them back to the caller and waits for the caller
laboratory over the telephone. What should the nurse do? to confirm that the nurse has understood the results. The nurse
may receive results by telephone; and although electronic transfer
to the client's medical record is appropriate, the nurse can also
accept the telephone results if the laboratory has called the results
to the nurses station.
offer to assist with the discharge teaching needs.

Staff members need to know the charge nurse is a supportive


Several hours into a shift, a nurse on a very busy medical-surgical
leader who respects their honesty and stands behind them. By
unit privately asks the charge nurse to change the nurse's assign-
offering to help with discharge teaching, the charge nurse is ac-
ment. The nurse is frustrated because so much time and energy
tively engaging with the staff at a time of need. Changing all the
has had to be devoted to helping a newly licensed nurse provide
assignments on this extremely busy floor would be counterpro-
discharge teaching for clients with diabetes mellitus. The charge
ductive. Insisting that the staff member follow through with their
nurse should
assignment disrespects the nurse's request and genuine need.
Providing a float nurse could help, but there are no guarantees a
float nurse is available.
Reduced serum ammonia levels.

The nurse administers lactulose to a client with cirrhosis. What is Lactulose is used to treat hepatic encephalopathy by reducing
the expected outcome from the administration of the lactulose? serum ammonia levels. It is not used to stimulate bowel peristalsis,
even though diarrhea can be a side effect of the drug. Lactulose
does not have any effect on edema, ascites, or hemorrhage.
"Hold your abdomen firmly with a pillow, and take several deep
breaths."

A client has an adrenal tumor and is scheduled for a bilateral Effective splinting for a high incision reduces stress on the in-
adrenalectomy. During preoperative teaching, the nurse teaches cision line, decreases pain, and increases the client's ability to
the client how to do deep-breathing exercises after surgery. What deep-breathe effectively. Deep breathing should be done hourly
should the nurse tell the client to do? by the client after surgery. Sitting upright ignores the need to splint
the incision to prevent pain. Tightening the stomach muscles is
not an effective strategy for promoting deep breathing. Raising the
shoulders is not a feature of deep-breathing exercises.
glucagon

During a hypoglycemic reaction, a layperson may administer


A nurse is teaching a client with type 1 diabetes how to treat
glucagon, an antihypoglycemic agent, to raise the blood glucose
adverse reactions to insulin. To reverse hypoglycemia, the client
level quickly in a client who can't ingest an oral carbohydrate.
ideally should ingest an oral carbohydrate. However, this treat-
Epinephrine isn't a treatment for hypoglycemia. Although 50%
ment isn't always possible or safe. Therefore, the nurse should
dextrose is used to treat hypoglycemia, it must be administered
advise the client to keep which alternate treatment on hand?
I.V. by a skilled healthcare professional. Hydrocortisone takes a
relatively long time to raise the blood glucose level and therefore
isn't effective in reversing hypoglycemia.
confusion and seizures

Classic signs of water intoxication include confusion and seizures,


Which findings indicate that a client has developed water intoxi-
both of which are caused by cerebral edema. Weight gain will also
cation secondary to treatment for diabetes insipidus?
occur. Sunken eyeballs, thirst, and increased BUN levels indicate
fluid volume deficit. Spasticity, flaccidity, and tetany are unrelated
to water intoxication.
3 / 27
Unit of 3: Endocrine & Metabolic Disorders
Study online at https://quizlet.com/_8igkej
preventing adrenal crisis

The priority in the first 24 hours after adrenalectomy is to identify


and prevent adrenal crisis. Monitoring of vital signs is the most im-
portant evaluation measure. Hypotension, tachycardia, orthostatic
A client has had an adrenalectomy. What is the priority goal for this hypotension, and arrhythmias can be indicators of pending vas-
client in the first 24 hours after surgery? cular collapse and hypovolemic shock that can occur with adrenal
crisis. Beginning oral nutrition is important, but not necessarily in
the first 24 hours after surgery, and it is not more important than
preventing adrenal crisis. Promoting self-care activities is not as
important as preventing adrenal crisis. Ambulating in the hallway
is not a priority in the first 24 hours after adrenalectomy.
with each meal and snack

In chronic pancreatitis, destruction of pancreatic tissue requires


Pancreatic enzyme replacements are prescribed for the client with
pancreatic enzyme replacement. Pancreatic enzymes are pre-
chronic pancreatitis. When should the nurse instruct the client to
scribed to facilitate the digestion of proteins and fats and should
take them to obtain the most therapeutic effect?
be taken in conjunction with every meal and snack. Specified
hours or limited times for administration are ineffective because
the enzymes must be taken in conjunction with food ingestion.
using sterile technique during the dressing change

The nurse should perform the dressing changes using sterile


technique to prevent infection. Applying heat should be avoided
Which intervention is essential when performing dressing
in a client with diabetes mellitus because of the risk of injury.
changes on a client with a diabetic foot ulcer?
Cleaning the wound with povidone-iodine solution and debriding
the wound with each dressing change prevents the development
of granulation tissue, which is essential in the wound healing
process.
at breakfast.
After a 3-month trial of dietary therapy, a client with type 2 diabetes
Like other oral antidiabetic agents ordered in a single daily dose,
still has blood glucose levels above 180 mg/dl (9.99mmol/L).
glyburide should be taken with breakfast. If the client takes gly-
The physician adds glyburide, 2.5 mg P.O. daily, to the treatment
buride later, such as in mid-morning, after dinner, or at bedtime,
regimen. The nurse should instruct the client to take the glyburide:
the drug won't provide adequate coverage for all meals consumed
during the day.
"I'll wear cotton socks with well-fitting shoes."

The client demonstrates an understanding of proper foot care


when stating they'll wear cotton socks with well-fitting shoes; cot-
ton socks wick moisture away from the skin, helping to prevent
The nurse teaches a client with diabetes mellitus about proper
fungal infections, and well-fitting shoes help avoid pressure areas.
foot care. Which statement indicates the client understands the
Aching isn't a common sign of foot problems; however, a tingling
teaching?
sensation in the feet indicates neurovascular changes. Clients with
diabetes should not soak feet unless specifically directed by a
healthcare provider as softening the skin may make it more prone
to injury. Although lotions are acceptable, the client should not
apply it between the toes, as this could promote a fungal infection.
hyperkalemia

Spironolactone is a potassium-sparing diuretic; therefore, clients


A client with cirrhosis begins to develop ascites. Spironolactone is should be monitored closely for hyperkalemia. Other common
prescribed to treat the ascites. The nurse should monitor the client adverse effects include abdominal cramping, diarrhea, dizziness,
closely for which drug-related adverse effect? headache, and rash. Constipation and dysuria are not common
adverse effects of spironolactone. An irregular pulse is not an
adverse effect of spironolactone but could develop if serum potas-
sium levels are not closely monitored.

4 / 27
Unit of 3: Endocrine & Metabolic Disorders
Study online at https://quizlet.com/_8igkej
Imbalanced nutrition: Less than body requirements related to thy-
roid hormone excess

In the client with hyperthyroidism, excessive thyroid hormone pro-


duction leads to hypermetabolism and increased nutrient metab-
olism. These conditions may result in a negative nitrogen balance,
increased protein synthesis and breakdown, decreased glucose
Which nursing diagnosis takes highest priority for a client with
tolerance, and fat mobilization and depletion. These changes put
hyperthyroidism?
the client at risk for marked nutrient and calorie deficiency, making
Imbalanced nutrition: Less than body requirements related to thy-
roid hormone excess the most important nursing diagnosis. Risk
for impaired skin integrity related to edema, skin fragility, and poor
wound healing and Disturbed body image related to weight gain
and edema may be appropriate for a client with hypothyroidism,
which slows the metabolic rate.
"Diabetes can affect sensation in your feet and you can hurt
yourself without realizing it."

The nurse should make the client aware that diabetes affects
When referred to a podiatrist, a client newly diagnosed with dia-
sensation in the feet and that they might hurt their foot but not
betes mellitus asks, "Why do you need to check my feet when I'm
feel the wound. Although it's important that the client's shoes fit
having a problem with my blood sugar?" The nurse's most helpful
properly, this isn't the only reason the client's feet need to be
response to this statement is
checked. Telling the client that diabetes mellitus increases the
risk of infection or stating that the circulation in the client's feet
indicates the severity of their diabetes doesn't provide the client
with complete information.
cortisol levels before and after the system is challenged with a
synthetic steroid
The nurse is caring for a client with possible Cushing's syndrome
undergoing diagnostic testing. The health care provider orders lab The dexamethasone suppression test measures cortisol levels
work and a dexamethasone suppression test. Which parameter before and after the system is challenged with a synthetic steroid.
would the nurse assess on the dexamethasone suppression test? The dexamethasone suppression test does not measure dexam-
ethasone or body chemicals altered in depression. Dexametha-
sone is used to challenge the cortisol level.
a client diagnosed with hypothyroidism and a heart rate of 48 beats
per minute

A heart rate of 48 beats per minute may have significant im-


The nurse is assigned to care for the following clients. Which client
plications for cardiac output and hemodynamic stability. Clients
should the nurse see first?
with Graves disease usually have a rapid heart rate, but 94 beats
per minute is a normal finding. The diabetic client may need
sliding-scale coverage, which is not urgent. Clients with Cushing
disease frequently have dependent edema.
Report the error, complete the proper paperwork, and meet with
the unit manager.
While reviewing the day's charts, a nurse who's been under a
Making an error can be very stressful and a nurse may feel great
great deal of personal stress realizes that the nurse forgot to
pressure to hide the mistake or not follow protocol. Discussing
administer insulin to client with diabetes mellitus. The nurse has
the problem with the unit coordinator may help the nurse address
made numerous errors in the past few weeks and is now afraid
some of the underlying stress that led up to making the error.
this job is in jeopardy. What is the bestcourse of action?
Nonetheless, the nurse must still report the error and complete
the proper paperwork. The nurse should contact the physician and
follow their instructions, but shouldn't bypass proper protocol.
The client will need steroid replacement for the rest of her life.

Bilateral adrenalectomy requires lifelong adrenal hormone re-


Which information should the nurse include in the teaching plan placement therapy. If unilateral surgery is performed, most clients
of a female client with bilateral adrenalectomy? gradually reestablish a normal secretion pattern. The client and
family will require extensive teaching and support to maintain
self-care management at home. Information on dosing, adverse
effects, what to do if a dose is missed, and follow-up examinations
5 / 27
Unit of 3: Endocrine & Metabolic Disorders
Study online at https://quizlet.com/_8igkej
is needed in the teaching plan. Although steroids are tapered
when given for an intermittent or one-time problem, they are not
discontinued when given to clients who have undergone bilateral
adrenalectomy because the clients will not regain the ability to
manufacture steroids. Steroids must be taken on a daily basis, not
just during periods of physical or emotional stress
Obtain the client's blood glucose at the bedside.

The nurse should first obtain the blood glucose level and then
contact the health care provider to clarify whether the client's usual
A client with type I diabetes mellitus is scheduled to have surgery.
insulin dose should be given before surgery; having the blood
The client has been nothing-by-mouth (NPO) since midnight. In
glucose level is objective information that the health care provider
the morning, the nurse notices the client's daily insulin has not
may need to know before making a final decision as to the insulin
been prescribed. Which action should the nurse do first?
dosage.
The nurse should not assume that the usual insulin dose is to be
given. It is not appropriate for the nurse to defer decision-making
on this issue until after the surgery.
lungs and kidneys

The lungs and kidneys facilitate the ratio of bicarbonate to carbonic


The nurse is caring for a client with multiple organ failure who
acid. Carbon dioxide is one of the components of carbonic acid.
is in metabolic acidosis. Which pair of organs is responsible for
The lungs regulate carbonic acid levels by releasing or conserving
regulatory processes and compensation?
CO2 by increasing or decreasing the respiratory rate. The kidneys
assist in acid-base balance by retaining or excreting bicarbonate
ions.
Tell the client that as the medication corrects the hormone defi-
ciency improvement in looks can be expected soon.

Stating that the client will soon experience improvement is sup-


A nurse is caring for a client with hypothyroidism. The client is portive and encouraging and offers direction in a way that mo-
extremely upset about altered physical appearance. The client tivates continued medication compliance. Stating that the client
doesn't want to take the medication because "it isn't doing any should ask the physician about the medication dosage might
good." What should the nurse do? cause the client alter the dosage, and also is putting the client off
instead of addressing the concerns. Stating that the client looks
fine discounts the client's feelings. Advising the client to practice
self-acceptance is parental and direct at a time when the client
needs support and understanding.
"No matter how much I drink, I'm still thirsty all the time."

Polydipsia, or increased thirst, is a classic clinical manifestation of


diabetes. The excessive loss of fluids is the result of the osmotic
When obtaining a health history from a client newly admitted to the
diuresis that occurs with glycosuria. Other clinical manifestations
hospital, which statement indicates the client's needs for further
include hunger, fatigue, blurred vision, slow-healing wounds, and
follow-up?
hyperglycemia. The report of shortness of breath is not an acute
issue and can be followed up upon later. In addition, painful joints
and having trouble urinating are a concern to the client and should
be addressed after the acute health concern is addressed.
acromegaly

Acromegaly, which is caused by a pituitary tumor that releases


A client with hyperglycemia who weighs 210 lb (95 kg) tells the
excessive growth hormone, is associated with hyperglycemia,
nurse that their spouse sleeps in another room because the
hypertension, diaphoresis, peripheral neuropathy, and joint pain.
client's snoring keeps the spouse awake. The nurse notices that
Enlarged hands and feet are related to lateral bone growth, which
the client has large hands and a hoarse voice. Which disorder
is seen in adults with this disorder. The accompanying soft tissue
would the nurse suspect as a possible cause of the client's hy-
swelling causes hoarseness and, commonly, sleep apnea. Type 1
perglycemia?
diabetes is usually seen in children, and newly diagnosed persons
are usually very ill and thin. Hypothyroidism and growth hormone
deficiency aren't associated with hyperglycemia.

6 / 27
Unit of 3: Endocrine & Metabolic Disorders
Study online at https://quizlet.com/_8igkej
Assess the client for hypoglycemia.

The nurse should first assess the client because a hypoglycemic


A client's 1200 blood glucose was inaccurately documented as reaction is likely to occur. At this time, the nurse also should
310 mg/dL (17.2 mmol/L) instead of 130 mg/dL (7.2 mmol/L). give the client a fast-acting simple carbohydrate. Then the nurse
This error was not noticed until 1300. The nurse administered should notify the HCP for prescriptions to prevent or treat severe
the sliding scale insulin for a blood glucose of 310 mg/dL (17.2 hypoglycemia. The nurse could consult the clinical pharmacist
mmol/L). What should the nurse do first? until able to contact the HCP, but the first action is to assess
the client in order to have accurate information to report. When
the situation has been resolved, the nurse should document the
incident and report the incident to the charge nurse.
an increase in the hypoglycemic effects of insulin.

There is a direct interaction between the effects of insulin and


A client with diabetes has been diagnosed with hypertension, and
those of beta blockers. The nurse must be aware that there is a
the health care provider has prescribed atenolol, a beta-blocker.
potential for increased hypoglycemic effects of insulin when a beta
When teaching the client about the drug, what should the nurse
blocker is added to the client's medication regimen. The client's
tell the client about how it may interact with the client's diabetes?
blood sugar should be monitored. Ketoacidosis occurs in hyper-
Atenol may cause:
glycemia. Although a decrease in the incidence of ketoacidosis
could occur when a beta blocker is added, the direct result is an
increase in the hypoglycemic effect of insulin.
"A person with diabetes should monitor their eating of proteins,
fats, and carbohydrates."

Diabetes mellitus is a multifactorial, systemic disease associated


with problems in the metabolism of all food types. The client's
The client with type 1 diabetes mellitus says, "If I could just avoid
diet should contain appropriate amounts of all three nutrients,
what you call carbohydrates in my diet, I guess I would be okay."
plus adequate minerals and vitamins. Limiting carbohydrate intake
What is the best response by the nurse?
is just part of a comprehensive diabetic diet plan. A client with
type 1 diabetes will need lifelong insulin therapy. Carbohydrates
from fruit and vegetable sources will still need to be factored into
carbohydrate intake. Telling a client "all we ask you to do" is a
value-judgement and is not therapeutic communication.
Have regular follow-up care.

The nurse should instruct the client with Graves' disease to have
regular follow-up care because most cases of Graves' disease
eventually result in hypothyroidism. Annual thyroid-stimulating
Which instruction should be included in the discharge teaching hormone tests and the client's ability to recognize signs and symp-
plan for a client after thyroidectomy for Graves' disease? toms of thyroid dysfunction will help detect thyroid abnormalities
early. Recording intake and output is important for clients with
fluid and electrolyte imbalances but not thyroid disorders. DDAVP
is used to treat diabetes insipidus. Although exercise to improve
cardiovascular fitness is important, the importance of regular fol-
low-up is most critical for this client.
anion gap

Metabolic acidosis is a common clinical disturbance characterized


by a low pH (increased H+concentration) and a low plasma bicar-
bonate concentration. It can be produced by a gain of hydrogen
ion or a loss of bicarbonate. It can be divided clinically into two
forms, according to the values of the serum anion gap: high anion
The nurse is caring for a client with a metabolic acidosis (pH 7.25).
gap acidosis and normal anion gap acidosis. A patient diagnosed
Which value is most useful to the nurse in determining whether the
with metabolic acidosis is determined to have normal anion gap
cause of the acidosis is due to acid gain or to bicarbonate loss?
metabolic acidosis if the anion gap is within this normal range. An
anion gap greater than 16 mEq (16 mmol/L) (the normal value
for an anion gap is 8-12 mEq/L (8-12 mmol/L) without potassium
in the equation. If potassium is included in the equation, the
normal value for the anion gap is 12-16 mEq/L (12-16 mmol/L) and
suggests an excessive accumulation of unmeasured anions and
would indicate high anion gap metabolic acidosis as the type. An
7 / 27
Unit of 3: Endocrine & Metabolic Disorders
Study online at https://quizlet.com/_8igkej
anion gap occurs because not all electrolytes are measured. More
anions are left unmeasured than cations. A low or negative anion
gap may be attributed to hypoproteinemia. Disorders that cause a
decreased or negative anion gap are less common compared to
those related to an increased or high anion gap.
"The wound nurse is specially trained to care for diabetic wounds."
A nurse is caring for a client with poorly managed diabetes mellitus
"You could possibly lose your foot without proper care."
who has a serious foot ulcer. When the nurse informs the client
"We're very concerned about your foot and we want to provide the
that the physician has ordered a wound care nurse to examine
best possible care for you."
the wound, the client asks why should anyone other than the staff
nurse care for the wound. The client states, "It's no big deal. I'll
Since diabetics are at an increased risk for loss of lower extremi-
keep it covered and put antibiotic ointment on it." Which responses
ties due to vascular problems, foot care specialists are warranted.
made by the nurse would be appropriate? Select all that apply.
Foot care nurses are specially trained to care for diabetic wounds.
I.V. normal saline and glucocorticoids
A client with a history of Addison's disease is experiencing weak-
The client with Addison's is expected to have hypotension and
ness and headache. The vital signs are blood pressure of 100/60
inadequate corticosteroids. There is no evidence that the client
and heart rate of 80. Laboratory values are Na 130, potassium
would be anemic. Although the blood pressure may be a little
4.8, and blood glucose 70. Which solution would the nurse expect
below normal, there is no indication for an inotropic drug such as
to administer?
dopamine to increase perfusion. There is no indication that the
client would be weak and hypoglcemic.
"What exactly do you mean by wanting 'everything' done for you?"

Asking the client what they mean is the best response. The nurse
should clarify the client's request and get as much information as
A client diagnosed with thyroid cancer signed a living will that possible before notifying the physician of the client's wishes. Ask-
states the client doesn't want ventilatory support if the condition ing the physician to revoke the client's do-not-resuscitate (DNR)
deteriorates. As the client's condition worsens, the client states, order makes an assumption about the client's wishes without
"I changed my mind. I want everything done for me." Which re- obtaining clarification of their statement. The client might want
sponse by the nurse is best? aggressive treatment without reversing the DNR order. Asking the
client if they understand that they'll be placed on a ventilator places
the client on the defensive. Telling the client to talk with family is
an inappropriate response; the client has the right to change their
treatment plan without input from their family.
serum sodium level of 124 mEq/L

In SIADH, the posterior pituitary gland produces excess antidi-


uretic hormone (vasopressin), which decreases water excretion by
A client is diagnosed with syndrome of inappropriate antidiuretic
the kidneys. This, in turn, reduces the serum sodium level, causing
hormone (SIADH). The nurse should anticipate which laboratory
hyponatremia, as indicated by a serum sodium level of 124 mEq/L.
test result?
In SIADH, the serum creatinine level isn't affected by the client's
fluid status and remains within normal limits. A hematocrit of 52%
and a BUN level of 8.6 mg/dl are elevated. Typically, the hematocrit
and BUN level decrease.
hydrocortisone
A client with a history of Addison's disease and flulike symp-
toms accompanied by nausea and vomiting over the past week Emergency treatment for acute adrenal insufficiency (addisonian
is brought to the facility. The client's spounse reports that the crisis) is I.V. infusion of hydrocortisone and saline solution. The
client acted confused and was extremely weak upon waking that client is usually given a dose containing hydrocortisone 100 mg
morning. The client's blood pressure is 90/58 mm Hg, pulse is 116 I.V. in normal saline every 6 hours until blood pressure returns
beats/minute, and temperature is 101° F (38.3° C). A diagnosis of to normal. Insulin isn't indicated in this situation because adrenal
acute adrenal insufficiency is made. What should the nurse expect insufficiency is usually associated with hypoglycemia. Potassium
to administer by I.V. infusion? isn't indicated because these clients are usually hyperkalemic. The
client needs normal — not hypotonic — saline solution.
metabolic acidosis

During surgery, a patient develops hypothermia. The circulating When a patient's temperature falls, glucose metabolism is re-
nurse would monitor the patient closely for which finding? duced. As a result, metabolic acidosis may develop. Rebound
hyperthermia, anaphylaxis, and alkalosis are not associated with
hypothermia during surgery.

8 / 27
Unit of 3: Endocrine & Metabolic Disorders
Study online at https://quizlet.com/_8igkej
Addison's disease
The nurse is caring for a client in the medical unit. The nurse
receives a health care provider's order for hydrocortisone 100 mg Intravenous hydrocortisone for clients in acute adrenal crisis is the
intravenously at a rate of 10 cc/hour for a client in acute adrenal proper treatment for individuals with Addison's disease. Cushing's
crisis. The nurse understands that this treatment is common in syndrome is associated with excessive amounts of glucocorti-
clients with which disease process? coids. Hyperthyroidism and hypoparathyroidism are not treated
with hydrocortisone.
take glipizide 30 minutes before breakfast.

Glipizide is most effective when taken 30 minutes before breakfast.


When teaching a client when to take glipizide in order to maximize
The duration of action is 10 to 24 hours.If the drug needs to be
the effectiveness of the drug, the nurse should instruct the client
taken more than once a day, the dosage may be divided and taken
to:
twice a day before meals.It is not as effective to take the drug after
meals.Although blood glucose levels will be monitored, the values
do not dictate when the drug should be taken.
pH: 7.55, PaCO2: 60 mm Hg, HCO3-: 28

The client's ABG would likely demonstrate metabolic alkalosis.


Metabolic alkalosis is a clinical disturbance characterized by a
Which of the following arterial blood gas (ABG) results would the high pH (decreased H+ concentration) and a high plasma bicar-
nurse anticipate for a client with a 3-day history of vomiting? bonate concentration. It can be produced by a gain of bicarbonate
or a loss of H+. A common cause of metabolic alkalosis is vomiting
or gastric suction with loss of hydrogen and chloride ions. The
disorder also occurs in pyloric stenosis where only gastric fluid is
lost. The other results do not represent metabolic alkalosis.
daily neck exercises

Surgical clients should be taught specific and general postoper-


ative care. Following thyroid surgery, clients should perform neck
exercises to maintain range of motion. Clients will be placed in
The nurse is teaching a client scheduled for thyroid lobectomy. semi-Fowler's position postoperatively to ease the work of breath-
What information will the nurse include? ing. All surgical clients should be taught to observe for common
surgical complications, such as bleeding or infection, and to pro-
vide postoperative wound care. After partial thyroidectomy for an
early-stage cancer, the client will not need a tracheostomy. If a
calcium imbalance develops, it will be hypocalcemia resulting from
removal or damage to the parathyroid glands.
Call the health care provider (HCP) for specific prescriptions
based on the glucose level.

The nurse should notify the HCP directly for specific prescriptions
On the day of surgery, a client with diabetes who takes insulin on a
based on the client's glucose level. The nurse cannot ignore the
sliding scale is to have nothing by mouth and all medications with-
elevated glucose level. The surgical experience is stressful, and
held. The client's 0600 glucose level is 300 mg/dL (16.7 mmol/L).
the client needs specific insulin coverage during the perioperative
What should the nurse do?
period. The nurse should not administer the insulin without check-
ing with the surgeon because there are specific prescriptions to
withhold all medications. It is not necessary to notify the surgery
department unless the HCP cancels the surgery.
sodium and potassium abnormalities.

In Addison's disease, a form of adrenocortical hypofunction, al-


dosterone secretion is reduced. Aldosterone promotes sodium
A client with Addison's disease comes to the clinic for a follow-up
conservation and potassium excretion. Therefore, aldosterone
visit. When assessing this client, the nurse should stay alert for
deficiency increases sodium excretion, predisposing the client
signs and symptoms of
to hyponatremia, and inhibits potassium excretion, predisposing
the client to hyperkalemia. Because aldosterone doesn't regulate
calcium, phosphorus, chloride, or magnesium, an aldosterone
deficiency doesn't affect levels of these electrolytes directly.

9 / 27
Unit of 3: Endocrine & Metabolic Disorders
Study online at https://quizlet.com/_8igkej
serum glucose level.

Hyperglycemia, which develops from glucocorticoid excess, is a


manifestation of Cushing's syndrome. With successful treatment
of the disorder, serum glucose levels decline. Hirsutism, not hair
A female client is being successfully treated for Cushing's syn-
loss, is common in Cushing's syndrome; therefore, with successful
drome. The nurse should expect a decline in
treatment, abnormal hair growth declines. Osteoporosis occurs in
Cushing's syndrome; therefore, with successful treatment, bone
mineralization increases. Amenorrhea develops in Cushing's syn-
drome. With successful treatment, the client experiences a return
of menstrual flow, not a decline in it.
"I can avoid getting sick by not becoming dehydrated and by
paying attention to my need to urinate, drink, or eat more than
usual."

Stating the need to remain hydrated and pay attention to eating,


A client diagnosed with hyperosmolar hyperglycemic nonketotic drinking, and voiding needs indicates that the client understands
syndrome (HHNS) is stabilized and prepared for discharge. When HHNS. Inadequate fluid intake during hyperglycemic episodes
preparing the client for discharge and home management, which commonly leads to HHNS. By recognizing the signs of hyper-
statement indicates that the client understands the condition and glycemia (polyuria, polydipsia, and polyphagia) and increasing
how to control it? fluid intake, the client may prevent HHNS. Drinking a glass of
nondiet soda would be appropriate for hypoglycemia. Limiting flu-
ids will exacerbate the development of HHNS; limiting food might
be acceptable, but it may lead to ketosis. A high-carbohydrate diet
would exacerbate the client's condition, particularly if fluid intake
is low.
Eat small meals more frequently.
Drink ginger tea.
Avoid fried foods.
A client with type 2 diabetes has just started to take dulaglutide.
The client reports having severe nausea. What should the nurse Nausea is a common side effect when clients first start taking
instruct the client to do to manage the nausea? Select all that dulaglutide. To manage the nausea the nurse can suggest that
apply. the client eat smaller meals more frequently, drink beverages with
ginger in them, and avoid fried foods. The client should decrease
the fat content in the diet. The client should not stop using the drug
unless prescribed by the health care provider.
Repeat important information during the presentation.
The nurse is educating a client on diabetes management. The
client is asking questions that cause the nurse to be concerned
Repetition is an effective means of reinforcing critical information
about the client's ability to retain the information. Which would be
and enhancing content retention. The other options will not in-
the best technique for the nurse to use to enhance the retention
crease the client's ability to retain information and may decrease
of information by the client?
the client's concentration and ability to retain critical information.
"I will be able to switch to insulin pills when my sugar is under
control."

When educating the client with type 1 diabetes, the nurse knows Oral antidiabetic agents are effective only in adult clients with
that more education is needed when the client says: type 2 diabetes. Oral antidiabetic agents aren't effective in type
1 diabetes. The need to eliminate sugar, give insulin, and receive
proper foot care are all items that indicate the client understands
the teaching.
both insulins 0.5 hours before breakfast
A client with newly diagnosed type 1 diabetes is scheduled to
receive regular insulin 10 units and NPH insulin 20 units every
Regular and NPH insulins are scheduled together one-half hour
morning. When should the nurse schedule the administration of
before breakfast. They do not need to be given separately or in
these medications?
different syringes.
Offer orange juice and crackers.

A client with type 1 diabetes who requires glucagon should be giv-


en a complex carbohydrate snack as soon as possible to restore
the liver glycogen and prevent secondary hypoglycemia. Orange
10 / 27
Unit of 3: Endocrine & Metabolic Disorders
Study online at https://quizlet.com/_8igkej
juice and crackers work well in treating hypoglycemia. Carbohy-
drates that contain fat, such as ice cream, are not recommended.
The nurse is caring for a client with type 1 diabetes. The nurse
The healthcare provider should be alerted to the hypoglycemic
finds the client unconscious and administers glucagon, 1 mg
event, but not until the client's blood sugar has stabilized. To
intramuscularly. What is the next action by the nurse when the
prevent further hypoglycemia, the nurse would not administer the
client regains consciousness?
ordered rapid-acting insulin until after the client's blood sugar has
normalized and the client is able to eat a meal.
hypopituitarism

Most clients who undergo adenoma removal experience a gradual


return of normal pituitary secretion and do not experience compli-
cations. However, hypopituitarism can cause growth hormone, go-
nadotropin, thyroid-stimulating hormone, and adrenocorticotropic
A client has had an hypophysectomy. What signs of a potential
hormone deficits. The client should be taught to monitor for change
complication should the nurse teach the client to report?
in mental status, energy level, muscle strength, and cognitive
function. In adults, changes in sexual function, impotence, or
decreased libido should be reported. Acromegaly and Cushing's
disease are conditions of hypersecretion. Diabetes mellitus is
related to the function of the pancreas and is not directly related
to the function of the pituitary.
vasopressin.

Because diabetes insipidus results from decreased antidiuretic


hormone (vasopressin) production, the nurse should expect to ad-
When caring for a client with diabetes insipidus, the nurse expects
minister synthetic vasopressin for hormone replacement therapy.
to administer
Furosemide, a diuretic, is contraindicated because a client with
diabetes insipidus experiences polyuria. Insulin and dextrose are
used to treat diabetes mellitus and its complications, not diabetes
insipidus.
potassium chloride

The nurse should question an order for potassium chloride be-


A nurse is caring for a client in addisonian crisis. Which medication cause addisonian crisis results in hyperkalemia. Administering
order should the nurse question? potassium chloride is contraindicated. Because the client is hy-
ponatremic, an order for normal saline solution is appropriate.
Hydrocortisone and fludrocortisone are used to replace deficient
adrenal cortex hormones.
assess vital signs.

Because the client in addisonian crisis is unstable, vital signs


and fluid and electrolyte balance should be assessed every 30
During the first 24 hours after a client is diagnosed with addisonian
minutes until the client is stable. Daily weights are sufficient when
crisis, which intervention should the nurse perform frequently?
assessing the client's condition. The client shouldn't have ketones
in their urine, so there is no need to assess the urine for their
presence. Oral hydrocortisone isn't administered during the first
24 hours in severe adrenal insufficiency.
blood glucose of 140 mg/dL (7.8 mmol/L)
An adult with type 2 diabetes mellitus has been NPO since 2200
in preparation for having a nephrectomy the next day. At 0600 on The client's blood glucose level is elevated, beyond levels accept-
the day of surgery, the nurse reviews the client's medical record ed for fasting; normal blood glucose range is 70 to 120 mg/dL (3.9
and laboratory results. Which finding should the nurse report to to 6.7 mmol/L). The specific gravity is within normal range (1.001
the health care provider (HCP)? to 1.030). Urine output should be 30 to 50 mL/h; thus, 350 mL is a
normal urinary output over 8 hours. The potassium level is normal.
Have the client void before the procedure.

Before paracentesis, the client is asked to void. This is done to


The nurse is preparing a client for paracentesis. What should the
collapse the bladder and decrease the risk of accidental blad-
nurse do?
der perforation. The abdomen is not prepared with an antiseptic
cleansing solution. The client is placed in a Fowler's position. The
client does not need to be put on NPO status before the procedure.

11 / 27
Unit of 3: Endocrine & Metabolic Disorders
Study online at https://quizlet.com/_8igkej
Increase carbohydrates and protein in the diet.

A low-fat, high-protein, high-carbohydrate diet is encouraged for a


client with hepatitis to promote liver rejuvenation. Nutrition intake is
The nurse is developing a teaching plan for the client with hepatitis
important because clients may be anorexic and experience weight
A. What should the nurse tell the client to do?
loss. Activity should be modified and adequate rest obtained to
promote recovery. Social isolation should be avoided, and edu-
cation on preventing transmission should be provided; the client
does not need to sleep in a separate room.
The body and mood will gradually return to normal.
The client who has undergone a bilateral adrenalectomy is con-
cerned about persistent body changes and unpredictable moods. As the body readjusts to normal cortisol levels, mood and physical
What should the nurse teach the client about these changes? changes will gradually return to a normal state. The body changes
are not permanent, and the mood swings should level off.
anxiety
activity intolerance

Diabetes insipidus is characterized by excessive output of dilute


urine. Common signs and symptoms include massive diuresis,
The nursing is caring for a newly admitted client with diabetes dehydration, and thirst. Additional findings include malaise, lethar-
insipidus. When forming the plan of care, which nursing diagnoses gy, and irritability. Nursing diagnoses that aim at providing inter-
are anticipated? Select all that apply. ventions to decrease the symptoms include Anxiety (irritability)
and activity intolerance (due to lethargy). The client has a fluid
volume deficit due to the excessive output of urine. Though the
client urinates frequently, there is no reason to believe that there
is an impaired physical mobility or self-care deficit. A client has
symptoms of hyperglycemia with diabetes mellitus.
Always wear socks, and preferably, shoes to protect the feet.
Check the feet daily to look for any injuries to the feet.
Use lotion on feet to keep skin from becoming dry and cracked.

A client with diabetes and peripheral neuropathy is being dis- The client with peripheral neuropathy has a risk for skin breakdown
charged from the hospital. What instruction should the nurse due to decreased sensation in lower legs and, particularly, the feet.
provide to decrease the risk for skin breakdown? Select all that The client should wear socks and shoes, check the feet daily, and
apply. apply lotion to moisten the dry skin, but lotion should not be applied
between the toes because lotion can cause skin maceration. The
client should use a nail file instead of clippers to prevent injury and
should not use a hot water bottle, as this can cause burns due to
the client's decreased sensation.
delayed wound healing

Persistent cortisol excess undermines the collagen matrix of the


skin, impairing wound healing. It also carries an increased risk of
A client has had a bilateral adrenalectomy. For which potential
infection and of bleeding. The wound should be observed and doc-
complication should the nurse assess the client?
umentation performed regarding the status of healing. Confusion
and emboli are not expected complications after adrenalectomy.
Malnutrition also is not an expected complication after adrenalec-
tomy. Nutritional status should be regained postoperatively.
lactated Ringer's solution

Lactated Ringer's solution, with an osmolality of approximately


273 mOsm/L, is isotonic. The nurse shouldn't give half-normal
A physician orders an isotonic I.V. solution for a client. Which
saline solution because it's hypotonic, with an osmolality of 154
solution should the nurse plan to administer?
mOsm/L. Giving 5% dextrose and normal saline solution (with an
osmolality of 559 mOsm/L) or 10% dextrose in water (with an
osmolality of 505 mOsm/L) also would be incorrect because these
solutions are hypertonic.
encouraging fluids.

The nurse should encourage fluid intake to prevent renal cal-


12 / 27
Unit of 3: Endocrine & Metabolic Disorders
Study online at https://quizlet.com/_8igkej
culi formation. Sodium should be encouraged to replace losses
When instructing a client diagnosed with hyperparathyroidism
in urine. Restricting potassium isn't necessary in hyperparathy-
about diet, the nurse should stress the importance of
roidism.
exacerbation
A client has been admitted to the hospital for the treatment of
This client has experienced a significant exacerbation of a chronic
diabetic ketoacidosis, a problem that was accompanied by a ran-
disease (diabetes mellitus), which has manifested as an acute
dom blood glucose reading of 31.9 mmol/L (575 mg/dL), vomiting,
threat to the client's health. Morbidity is an epidemiological statistic
and shortness of breath. This client has experienced which of the
of the frequency of a disease. The client's problem does not have
following phenomena?
an infectious etiology and while risk factors underlie the present
condition, they are not the essence of the current state.
"It's so hard to predict when this scar will disappear."
A 24-year-old client with diabetes mellitus sustains a large lac-
eration that requires suturing. Which statement indicates that the In a client with diabetes, wound healing is delayed and unable to
client understands wound healing? be predicted. A specific time frame for healing is unrealistic as is
the statement that suturing does not produce a scar.
as potent as morphine in larger doses.

Hydromorphone hydrochloride is about five times more potent


A client undergoing a bilateral adrenalectomy has postoperative
than morphine sulfate, from which it is prepared. Therefore, it is
prescriptions for hydromorphone hydrochloride 2 mg to be ad-
administered only in small doses. Hydromorphone hydrochloride
ministered subcutaneously every 4 hours as needed for pain. Why
can cause dependency in any dose; however, fear of dependency
should the nurse administer hydromorphone in small doses?
developing in the postoperative period is unwarranted. The dose
A small dose is:
is determined by the client's need for pain relief. Hydromorphone
hydrochloride is not irritating to subcutaneous tissues. As with
opioid analgesics, excretion depends on normal liver function.
"Test your blood glucose every 4 hours."

The nurse should instruct a client with diabetes mellitus to check


their blood glucose levels every 3 to 4 hours and take insulin or
Which instruction should a nurse give to a client with diabetes
an oral antidiabetic agent as usual, even when the client is sick.
mellitus when teaching about "sick day rules"?
If the client's blood glucose level rises above 300 mg/dl, the client
should call their physician immediately. If the client is unable to
follow the regular meal plan because of nausea, the client should
substitute soft foods, such as gelatin, soup, and custard.
serum potassium level of 5.8 mEq/L (5.8 mmol/L)

Addison's disease decreases the production of aldosterone, cor-


tisol, and androgen, causing urinary sodium and fluid losses, an
A client is admitted to the health care facility for evaluation for
increased serum potassium level, and hypoglycemia. Therefore,
Addison's disease. Which laboratory test result best supports a
an elevated serum potassium level of 5.8 mEq/L best supports a
diagnosis of Addison's disease?
diagnosis of Addison's disease. A BUN level of 12 mg/dl and a
blood glucose level of 90 mg/dl are within normal limits. In a client
with Addison's disease, the serum sodium level would be much
lower than 134 mEq/L, a nearly normal level.
Insulin is absorbed more rapidly at abdominal injection sites than
at other sites.
A nurse explains to a client that the nurse will administer the
client's first insulin dose in the client's abdomen. How does ab-
Subcutaneous insulin is absorbed most rapidly at abdominal in-
sorption at the abdominal site compare with absorption at other
jection sites, more slowly at sites on the arms, and slowest at sites
sites?
on the anterior thigh. Absorption after injection in the buttocks is
less predictable.
Request information about nursing responsibilities in caring for a
client with a pump.
A nurse is assigned to a client who is using an insulin pump. The
Taking the initiative to gain new information relevant to client
nurse has never cared for a client with an insulin pump and isn't
care as well as expressing a desire to support the unit's needs
sure what to do. What should the nurse do first?
is an appropriate and professional nursing response. Refusing
the assignment is inappropriate because the nurse isn't taking
any initiative to learn about the pump. Refusing to care for the
13 / 27
Unit of 3: Endocrine & Metabolic Disorders
Study online at https://quizlet.com/_8igkej
client until the nurse receives training is inappropriate; the nurse
should gather information and evaluate the client before refusing
to provide care. Accepting the assignment doesn't address the
issue of lack of knowledge and may put the nurse or the client in
jeopardy.
cerebral edema

Noncompliance with treatment for SIADH may lead to water in-


toxication from fluid retention caused by excessive antidiuretic
A client is diagnosed with syndrome of inappropriate antidiuretic
hormone. This, in turn, limits water excretion and increases the
hormone (SIADH). The nurse informs the client that the physician
risk for cerebral edema. Hypovolemic shock results from, severe
will order diuretic therapy and restrict fluid and sodium intake to
deficient fluid volume; in contrast, SIADH causes excess fluid
treat the disorder. If the client doesn't comply with the recom-
volume. The major electrolyte disturbance in SIADH is dilutional
mended treatment, which complication may arise?
hyponatremia, not hyperkalemia. Because SIADH doesn't alter
renal function, potassium excretion remains normal; therefore,
severe hyperkalemia doesn't occur. Tetany results from hypocal-
cemia, an electrolyte disturbance not associated with SIADH.
vasopressin.
A nurse is caring for a client with diabetes insipidus. The nurse
Vasopressin is given subcutaneously to manage diabetes in-
should anticipate administering
sipidus. Insulin is used to manage diabetes mellitus. Furosemide
causes diuresis. Potassium chloride is given for hypokalemia.
methimazole
A client has been diagnosed with hyperthyroidism and presents
Methimazole is the drug of choice for this client. Radioactive
with heat intolerance and a blood pressure of 174/70 mm Hg; she
iodine is usually used for hyperthyroidism but is contraindicated in
is 3 months pregnant. The nurse anticipates that the physician will
pregnancy. Levothyroxine sodium is for hypothyroidism. Lisinopril
order which medication?
is an ACE inhibitor (ACEI) — all ACEIs are contraindicated in
pregnancy.
hyperparathyroidism

Hyperparathyroidism is characterized by osteopenia and renal


The nurse is caring for a client on the urinary unit. When providing
calculi secondary to overproduction of parathyroid hormone. The
report to the next shift, it is noted that the client has osteopenia and
hallmark symptom of hypoparathyroidism is tetany from hypocal-
history of renal calculi. Which disorder would the nurse suspect?
cemia. Hypopituitarism presents with extreme weight loss and
atrophy of all endocrine glands. Symptoms of hypothyroidism in-
clude hair loss, weight gain, and cold intolerance.
calcium gluconate

The client with tetany is suffering from hypocalcemia, which is


Which medication should be available to provide emergency treated by administering an IV preparation of calcium, such as
treatment if a client develops tetany after a subtotal thyroidecto- calcium gluconate or calcium chloride. Oral calcium is then neces-
my? sary until normal parathyroid function returns. Sodium phosphate
is a laxative. Echothiophate iodide is an eye preparation used as a
miotic for an antiglaucoma effect. Sodium bicarbonate is a potent
systemic antacid.
decreasing contact with blood and blood-contaminated fluids

Hepatitis C is usually transmitted through blood exposure or


needlesticks. A hepatitis C vaccine is currently under develop-
The nurse should institute which measure to prevent transmission
ment, but it is not available for use. The first line of defense against
of the hepatitis C virus to health care personnel?
hepatitis B is the hepatitis B vaccine. Hepatitis C is not transmitted
through feces or urine. Wearing a gown and mask will not prevent
transmission of the hepatitis C virus if the caregiver comes in
contact with infected blood or needles.
Measure and record urinary output.

Diabetes insipidus is characterized by polyuria (up to 8 L/day),


constant thirst, and an unusually high oral intake of fluids. Treat-
ment with the appropriate drug should decrease both oral fluid
14 / 27
Unit of 3: Endocrine & Metabolic Disorders
Study online at https://quizlet.com/_8igkej
The nurse is caring for a client following a motor vehicle incident intake and urine output. A urine output of 200 ml/hour indicates
with head trauma suspected of diabetes insipidus. Which nursing continuing polyuria. Blood sugar has nothing to do with diabetes
intervention is appropriate? insipidus.
primes the pen by expelling any air
injects the insulin in sites around the abdomen
stores the unopened pens in the refrigerator

A client is to use an insulin pen. Which action indicates the client Insulin pens should be stored in the refrigerator before use; once
is using the pen correctly? Select all that apply. opened they can be stored at a cool room temperature. The pen
needs to be primed by expelling air before injecting the insulin.
After the injection, the site can be patted, but not massaged.
Needles cannot be reused; the client should remove the needle
and place in a hard plastic container for disposal.
tell the physician the client isn't comfortable consenting to surgery
at this point.

The nurse has evaluated the client's knowledge concerning the


When a nurse attempts to make sure the physician obtained surgery and determined that the client doesn't have enough infor-
informed consent for a thyroidectomy, the nurse realizes the client mation to give informed consent. Even though the physician might
doesn't fully understand the surgery. The nurse approaches the want to move ahead, the nurse should advocate for the client by
physician, who curtly says, "I've told this client all about it. Just get telling the physician the client isn't ready for the surgery. Telling
the consent." The nurse should the physician that the client hasn't been given enough information
would be rude. The nurse shouldn't ask the charge nurse to talk
with the physician unless the physician refuses to accept the
nurse's professional opinion. Explaining surgery for the purpose
of obtaining consent is beyond the nurse's scope of practice.
"There is a strong link between alcohol use and acute pancreati-
tis."

Alcoholism is a major cause of acute pancreatitis in the United


States and Canada. Because some clients are reluctant to discuss
alcohol use, staff may inquire about it in several ways. Generally,
The client who has been hospitalized with pancreatitis does not
alcohol intake does not interfere with the tests used to diagnose
drink alcohol because of religious convictions. The client becomes
pancreatitis. Recent ingestion of large amounts of alcohol, howev-
upset when the health care provider (HCP) persists in asking
er, may cause an increased serum amylase level. Large amounts
about alcohol intake. What should the nurse tell the client about
of ethyl and methyl alcohol may produce an elevated urinary
the reason for these questions?
amylase concentration. All clients are asked about alcohol and
drug use on hospital admission, but this information is especially
pertinent for clients with pancreatitis. HCPs do need to seek facts,
but this can be done while respecting the client's religious beliefs.
Respecting religious beliefs is important in providing holistic client
care.
Document presenting signs and symptoms.
Maintain intake and output records.
Compare ABG findings with previous results.

Metabolic alkalosis results in increased plasma pH because of


The physician has prescribed sodium chloride for a hospitalized accumulated base bicarbonate or decreased hydrogen ion con-
51-year-old client in metabolic alkalosis. Which nursing actions centrations. The result is retention of sodium bicarbonate and
are required to manage this client? Select all that apply. increased base bicarbonate. Nursing management includes doc-
umenting all presenting signs and symptoms to provide accu-
rate baseline data, monitoring laboratory values, comparing ABG
findings with previous results (if any), maintaining accurate intake
and output records to monitor fluid status, and implementing pre-
scribed medical therapy.

blood pressure that has decreased from baseline


Which assessment finding requires immediate action in a client
A decreasing blood pressure may indicate that the client is expe-
diagnosed with Addison's disease?
riencing an Addisonian crisis. Tachycardia and tachypnea are also
manifestations of an Addisonian crisis. Fever can be an indication

15 / 27
Unit of 3: Endocrine & Metabolic Disorders
Study online at https://quizlet.com/_8igkej
of Addisonian crisis, but the temperature has only increased one
degree.
A client with type 1 diabetes takes 15 units of insulin isophane salicylate-containing preparations
before breakfast and 8 units before dinner. During a follow-up
visit, the nurse reevaluates the client's knowledge about insulin The client requires additional teaching if they take salicylates with
therapy and self-administration skills. The nurse realizes the client insulin. Salicylates may interact with insulin causing hypoglycemia.
requires additional teaching when the nurse discovers the client Antacids, vitamins with iron, and acetaminophen aren't known to
takes which over-the-counter preparations? interact with insulin.
"Various circumstances increase the need for glucocorticoids, so
I will need to adjust the dosage."

The need for glucocorticoids changes with circumstances. The


basal dose is established when the client is discharged, but this
dose covers only normal daily needs and does not provide for
additional stressors. As the manager of the medication schedule,
The client with Addison's disease is taking glucocorticoids at
the client needs to know signs and symptoms of excessive and
home. Which statement indicates that the client understands how
insufficient dosages. Glucocorticoid needs fluctuate. Glucocor-
to take the medication?
ticoids are not cumulative and must be taken daily. They must
never be discontinued suddenly; in the absence of endogenous
production, Addisonian crisis could result. Two-thirds of the daily
dose should be taken at about 0800 and the remainder at about
1600. This schedule approximates the diurnal pattern of normal
secretion, with highest levels between 0400 and 0600 and lowest
levels in the evening.
jugular vein distention and confusion

SIADH results in antidiuretic hormone (ADH) overproduction,


which leads to fluid retention and dilutional hyponatremia. Severe
SIADH can cause such complications as vascular fluid overload,
The nurse is assessing a client with the syndrome of inappropriate signaled by jugular vein distention. Hyponatremia results in os-
antidiuretic hormone secretion (SIADH). What findings does the motic fluid shifts in the brain that lead to neurological changes
nurse attribute to complications of this condition? such as irritability and confusion. Tetany and laryngeal spasms
are associated with hypocalcemia. Thirst is associated with hy-
pernatremia. Weight gain would be expected with fluid retention.
Cardiac arrhythmia would be seen with abnormal potassium lev-
els. Polyuria would be associated with diabetes insipidus, a lack
of ADH.
"If I follow my diet and exercise, I won't have diabetes any more."
"I can never eat a hot fudge sundae again."
"I guess I will need to stop meeting my friends at the coffee shop."

Patients with type 2 diabetes who follow a diet and exercise


An obese client, age 65, is diagnosed with type 2 diabetes. When
program will likely be able to achieve normal blood sugar levels,
educating this client about the diagnosis, the nurse knows that
but cannot consider themselves "cured" of diabetes. Renal failure
more education is needed when the client says which statement?
is a possible complication of uncontrolled diabetes. People with
Select all that apply.
well controlled diabetes can modify their diet to include occasional
treats like ice cream if they select sugar free versions. Meeting
friends for coffee is fine as long as the client does not include high
sugar items along with the beverage. Type 2 diabetes can often be
controlled with oral hypoglycemics.
A nurse is caring for a client who is prescribed 1 unit of regular
7
insulin for every 6 g of carbohydrates consumed at mealtime and
1 unit of regular insulin for every 30 mg/dL increase of blood sugar
According to the guidelines, 30 g carbohydrates/6 = 5 units regular
above 130 mg/dL. For lunch, the client ate a hamburger bun (20 g
insulin; blood sugar 190-130 = 60 mg above 1 unit for each 30
carbohydrates), a 3-oz burger (10 g carbohydrates), and a 12-oz
mg/dl elevation = 2; 5 + 2 = 7 units of regular insulin. It is important
diet soda (0 g carbohydrates). The client's blood sugar is now 190
to calculate the required insulin to be given as a corrective dose
mg/dL. How much total insulin will the nurse administer? Record
with each meal.
your answer using a whole number.

amount
odor
16 / 27
Unit of 3: Endocrine & Metabolic Disorders
Study online at https://quizlet.com/_8igkej
glucose level
ketone bodies
When reviewing the urinalysis report of a client with newly di-
agnosed diabetes mellitus, the nurse would expect which urine
Diabetes mellitus is associated with increased amounts of urine,
characteristics to be abnormal? Select all that apply.
a sweet or fruity odor, and glucose and ketone bodies in the urine.
It does not affect the urine's pH or specific gravity.
having wisdom teeth extracted

The nurse is instructing a college student with Addison's disease Adrenal crisis can occur with physical stress, such as surgery,
how to adjust the dose of glucocorticoids. The nurse should ex- dental work, infection, flu, trauma, and pregnancy. In these sit-
plain that the client may need an increased dosage of glucocorti- uations, glucocorticoid and mineralocorticoid dosages are in-
coids in which situation? creased. Weight loss, not gain, occurs with adrenal insufficiency.
Psychological stress has less effect on corticosteroid need than
physical stress.
"Your child will need less blood work as their glucose levels stabi-
lize."

Telling the parents that the number of blood draws will decrease as
their child's glucose levels stabilize engages them in the learning
A physician orders blood glucose levels every 4 hours for a
process and gives them hope that the present discomfort will end
4-year-old child with brittle type 1 diabetes. The parents are wor-
as the child's condition improves. Telling the parents that their child
ried that drawing so much blood will traumatize their child. How
won't remember the experience disregards their concerns and
can the nurse best reassure the parents?
anxiety. The nurse shouldn't offer to ask the physician to reduce
the number of blood draws; the physician needs the laboratory
results to monitor the child's condition properly. Although telling
the parents that the laboratory technicians are gentle and use tiny
needles may be reassuring, it isn't the most appropriate response.
Administer 1 ampule of 50% dextrose solution IV.

The client with a decreased level of consciousness and a fin-


gerstick glucose result of 39 mg/dL is experiencing dangerous
hypoglycemia. The nurse would first administer 50% dextrose
A client with type 1 diabetes presents with a decreased level of
solution IV to restore the client's physiological integrity. Because
consciousness and a fingerstick glucose level of 39 mg/dl (2.2
of the decreased level of consciousness, offering the client orange
mmol/L). A family member reports the client has been skipping
juice and crackers would put the client at risk for aspiration. Re-
meals in an effort to lose weight. What is the next action by the
ferring the client to a diabetes educator is important to address
nurse?
the methods of weight loss, but this would be appropriate after
the hypoglycemia is treated. The nurse would treat the dangerous
hypoglycemia with the ordered dextrose solution before contacting
a healthcare provider to prevent further complications of hypo-
glycemia.
related to bone demineralization resulting in pathologic fractures

Poorly controlled hyperparathyroidism may cause an elevated


A client with a history of chronic hyperparathyroidism admits to serum calcium level. This increase, in turn, may diminish calcium
being noncompliant. Based on initial assessment findings, the stores in the bone, causing bone demineralization and setting the
nurse formulates the nursing diagnosis of Risk for injury. To com- stage for pathologic fractures and a risk for injury. Hyperparathy-
plete the nursing diagnosis statement for this client, which "relat- roidism doesn't accelerate the metabolic rate. A decreased thyroid
ed-to" phrase should the nurse add? hormone level, not an increased parathyroid hormone level, may
cause edema and dry skin secondary to fluid infiltration into the
interstitial spaces. Hyperparathyroidism causes hypercalcemia,
not hypocalcemia; therefore, it isn't associated with tetany.
weight gain, decreased appetite, and constipation

Hashimoto's thyroiditis, an autoimmune disorder, is the most com-


mon cause of hypothyroidism. It's seen most frequently in women
Which findings should a nurse expect to assess in client with
older than age 40. Signs and symptoms include weight gain, de-
Hashimoto's thyroiditis?
creased appetite; constipation; lethargy; dry cool skin; brittle nails;
coarse hair; muscle cramps; weakness; and sleep apnea. Weight
loss, increased appetite, and hyperdefecation are characteristic of
hyperthyroidism. Weight loss, increased urination, and increased
17 / 27
Unit of 3: Endocrine & Metabolic Disorders
Study online at https://quizlet.com/_8igkej
thirst are characteristic of uncontrolled diabetes mellitus. Weight
gain, increased urination, and purplish-red striae are characteris-
tic of hypercortisolism.
restricting fluids to 800 ml/day

Excessive release of antidiuretic hormone (ADH) disturbs fluid


and electrolyte balance in SIADH. The excessive ADH causes
an inability to excrete dilute urine, retention of free water, expan-
A client is diagnosed with syndrome of inappropriate antidiuretic
sion of extracellular fluid volume, and hyponatremia. Symptomatic
hormone (SIADH). Laboratory results reveal serum sodium level
treatment begins with restricting fluids to 800 ml/day. Vasopressin
130 mEq/L and urine specific gravity 1.030. Which nursing inter-
is administered to clients with diabetes insipidus a condition in
vention helps prevent complications associated with SIADH?
which circulating ADH is deficient. Elevating the head of the bed
decreases vascular return and decreases atrial-filling pressure,
which increases ADH secretion, thus worsening the client's con-
dition. The client's sodium is low and, therefore, shouldn't be
restricted.
rapid, thready pulse

This client's abnormally high blood glucose level indicates hy-


A client who was diagnosed with type 1 diabetes 14 years ago
perglycemia, which typically causes polyuria, polyphagia, and
is admitted to the medical-surgical unit with abdominal pain. On
polydipsia. Because polyuria leads to fluid loss, the nurse should
admission, the client's blood glucose level is 470 mg/dl (26.1
expect to assess signs of deficient fluid volume, such as a rapid,
mmol/L). Which finding is most likely to accompany this blood
thready pulse; decreased blood pressure; and rapid respirations.
glucose level?
Cool, moist skin and arm and leg trembling are associated with
hypoglycemia. Rapid respirations — not slow, shallow ones — are
associated with hyperglycemia.
Hypoglycemia.

The peak action of regular insulin is approximately 2 to 3 hours af-


The nurse has been assigned to a client who has had diabetes
ter administration. The client is having typical hypoglycemic symp-
for 10 years. The nurse gives the client's usual dose of regular
toms. Acidosis results from uncontrolled diabetes mellitus, with
insulin at 7 a.m. At 10:30 a.m., the client has light-headedness
hyperpnea (Kussmaul respirations) as the outstanding symptom.
and sweating. The nurse should contact the physician, report the
The hallmark symptoms of hyperglycemia are increased thirst,
situation, background, and assessment, and recommend inter-
fruity breath, and glycosuria. The signs and symptoms of diabetic
vention for:
ketoacidosis include Kussmaul respirations, fruity breath, tachy-
cardia, abdominal pain, nausea, vomiting, headache, thirst, dry
skin, and dehydration.
Observe stools for steatorrhea.

If the dosage and administration of pancreatic enzymes are ade-


quate, the client's stool will be relatively normal. Any increase in
The client has chronic pancreatitis. What should the nurse teach
odor or fat content would indicate the need for dosage adjustment.
the client to do to monitor the effectiveness of pancreatic enzyme
Stable body weight would be another indirect indicator. Fluid in-
replacement?
take does not affect enzyme replacement therapy. If diabetes has
developed, the client will need to monitor glucose levels. However,
glucose and ketone levels are not affected by pancreatic enzyme
therapy and would not indicate effectiveness of the therapy.
maintain normal fluid and electrolyte balance.

Which statement indicates that the client with diabetes insipidus Because diabetes insipidus involves excretion of large amounts of
understands how to manage care? fluid, maintaining normal fluid and electrolyte balance is a priority
The client will: for this client. Special dietary programs or restrictions are not
indicated in treatment of diabetes insipidus. Serum glucose levels
are priorities in diabetes mellitus but not in diabetes insipidus.
pH 7.48

When evaluating a client's arterial blood gases (ABGs), which Metabolic alkalosis is a clinical disturbance characterized by a
value is consistent with metabolic alkalosis? high pH and high plasma bicarbonate concentration. The HCO3
value is below normal. The PaCO2 value and the oxygen satura-
tion level are within a normal range.

18 / 27
Unit of 3: Endocrine & Metabolic Disorders
Study online at https://quizlet.com/_8igkej
"I will use salt substitute to flavor my foods."

The Addison's client will have high potassium, low sodium, and
low calcium and exhibit hyperpigmentation due to the deficit of
A client is diagnosed with Addison's disease. Which statement by corticosteroids. Using a salt substitute requires further instruction,
the client to the nurse would require further instruction? as salt substitutes contain potassium. The client with Addison's
disease has high levels of potassium. Steroids tend to cause
stomach distress, so it is appropriate to take with food to de-
crease these symptoms. Increasing calcium is encouraged, and
sunscreen is appropriate due to the hyperpigmentation of the skin.
"I'll set up a meeting for today. Then you and I can meet to talk
about how things went."

he client may feel overwhelmed and anxious about this diagnosis.


The client's made a therapeutic connection with the nurse at a
A physician has referred a client newly diagnosed with diabetes
vulnerable time in their life when the client must address many
mellitus to the diabetes nurse-educator. When the nurse brings up
new issues. Offering to follow up with the client encourages them
the subject, the client states, "I'd rather work with you than with a
to move forward and gives them an opportunity to meet with
stranger." What is the nurse's best response?
a safe and trusted person afterward. Telling the client that the
nurse-educator is more knowledgeable about the subject doesn't
help address the client's feelings. Telling the client not to worry or
that they'll get over these feelings minimizes the client's feelings
and may impair the nurse-client relationship.
sore throat

The most serious adverse effects of PTU are leukopenia and


agranulocytosis, which usually occur within the first 3 months of
treatment. The client should be taught to promptly report to the
Propylthiouracil (PTU) is prescribed for a client with Graves' dis-
health care provider (HCP) signs and symptoms of infection, such
ease. Which symptom should the nurse teach the client to report?
as a sore throat and fever. Clients having a sore throat and fever
should have an immediate white blood cell count and differential
performed, and the drug must be withheld until the results are
obtained. Painful menstruation, constipation, and increased urine
output are not associated with PTU therapy.
nervousness

The four most commonly reported signs and symptoms of hy-


poglycemia are nervousness, weakness, perspiration, and con-
The nurse should teach the diabetic client that which is most
fusion. Other signs and symptoms include hunger, incoherent
indicative of hypoglycemia?
speech, tachycardia, and blurred vision. Anorexia and Kuss-
maul's respirations are clinical manifestations of hyperglycemia
or ketoacidosis. Bradycardia is not associated with hypoglycemia;
tachycardia is.
hypoglycemia.

Chlorpropamide is an antidiabetic agent. Clients should be ob-


The nurse should assess a client taking chlorpropamide for: served for signs and symptoms of hypoglycemia. Other com-
mon side effects include anorexia, nausea, vomiting, and heart-
burn.The drug does not cause dumping syndrome, oral candidia-
sis, or extrapyramidal symptoms.
Encircle the client's neck with both hands, have the client slightly
extend their neck, and ask them to swallow.

When palpating the thyroid gland, the nurse should encircle the
A nurse is preparing to palpate a client's thyroid gland. Which client's neck with both hands, have the client slightly extend their
action by the nurse is appropriate? neck, and ask them to swallow. As the client swallows, the gland is
palpated for enlargement as the tissue rises and falls. Having the
client flex their neck wouldn't allow for palpation. Massaging the
area or checking during inhalation doesn't allow for the movement
of tissue that swallowing provides.

19 / 27
Unit of 3: Endocrine & Metabolic Disorders
Study online at https://quizlet.com/_8igkej
dietary fiber

Foods high in dietary fiber tend to blunt the rise in blood glucose
levels after meals. Dietary fiber is the part of food not broken down
and absorbed during digestion. Most fibers come from plants;
The nurse teaches the client with type 1 diabetes mellitus about
good sources include whole grains, legumes, vegetables, fruits,
the importance of maintaining stable blood glucose levels. The
and nuts. The other foods do not minimize this rise in blood
nurse should suggest the client include which type of food to
sugar after meals. Dairy products are poor sources of fiber. Foods
minimize the rise in blood glucose level after meals?
fortified with vitamins are satisfactory if they also contain fiber.
However, many foods fortified with vitamins contain either no
dietary fiber (such as fortified milk) or little fiber (such as products
fortified with vitamins but made with refined grains). Meats are
poor sources of fiber.
Ask the physician to delay the discharge because the client re-
quires further teaching
Ask the physician for a referral for a diabetes nurse-educator to
see the client before discharge.

The nurse's primary concern should be the safety of the client after
A nurse has been caring for a client newly diagnosed with diabetes discharge. The nurse should provide succinct information to the
mellitus. The client is overwhelmed by the diagnosis and not sure physician concerning the client's needs; express concern about
about injecting insulin. This client has been discharged and the ensuring the client's safety; and ask the physician to delay the
charge nurse is insisting the nurse hurry because the space is client's discharge, and to provide a referral for the diabetes nurse
needed for clients being admitted. How should the nurse handle educator. The nurse should not suggest that the client rely on a
the situation? Select all that apply. friend or family member because the nurse doesn't know if a friend
or family member will be available to help. Telling the charge nurse
to do the education is not appropriate because the charge nurse
has too many other obligations to be able to effectively offer the
client thorough education. Offering the unit phone number is not
a good option because the client shouldn't be discharged until the
client can safely carry out aftercare.
"I'll get the chart and set up a time for you to review it with your
healthcare provider."

Every client has a right to access information that the hospital has
collected about the client. However, it is in the client's best interests
to have a knowledgeable professional present to explain compli-
A client recently diagnosed with hyperparathyroidism demands to
cated information and unfamiliar terminology that the chart might
see what the healthcare provider has written in the chart. What is
include. Having the client sign a release of medical information
the nurse's best response?
may be necessary, but that does not assist the client to schedule
a review with the healthcare provider. Suggesting the client review
the chart with the healthcare provider does not facilitate the review.
Contacting medical records to set up a time for the client to review
does not ensure that a knowledgable professional is available to
assist the client during the review.
15 g of a simple carbohydrate.

To reverse hypoglycemia, the American Diabetes Association


An agitated, confused client arrives in the emergency depart- (Canadian Diabetes Association) guidelines recommend ingest-
ment. The client's history includes type 1 diabetes, hyperten- ing 15 g of a simple carbohydrate, such as 15 g of glucose tablets,
sion, and angina pectoris. Assessment reveals pallor, diaphore- 3 teaspoons (15 mL) or 3 packets of table sugar dissolved in water,
sis, headache, and intense hunger. A stat blood glucose sample 3/4 cup (175 mL) of juice or regular soft drink, 6 LifeSavers (1 = 2.5
measures 42 mg/dl, (2.3 mmol/L) and the client is treated for an g carbohydrate), or a 1 tablespoon (5 mL) of honey. Then the client
acute hypoglycemic reaction. After recovery, the nurse teaches should check their blood glucose after 15 minutes. If necessary,
the client to treat hypoglycemia by ingesting this treatment may be repeated in 15 minutes. Ingesting only 2 to
5 g of a simple carbohydrate may not raise the blood glucose level
sufficiently. Ingesting more than 15 g may raise it above normal,
causing hyperglycemia.
profound neuromuscular irritability.

Hypoparathyroidism may slow bone resorption, reduce the serum


20 / 27
Unit of 3: Endocrine & Metabolic Disorders
Study online at https://quizlet.com/_8igkej
calcium level, and cause profound neuromuscular irritability (as
A client is admitted to an acute care facility with a tentative diag- evidenced by tetany). Hypoparathyroidism doesn't alter blood
nosis of hypoparathyroidism. The nurse should monitor the client pressure or affect the thirst mechanism, which usually is triggered
closely for the related problem of by fluid volume deficit. Gastritis doesn't cause or result from hy-
poparathyroidism.
tetany

Tetany may result if the parathyroid glands are excised or dam-


A nurse is assessing a client after a thyroidectomy. The assess- aged during thyroid surgery. Hemorrhage is a potential compli-
ment reveals muscle twitching and tingling, along with numbness cation after thyroid surgery but is characterized by tachycardia,
in the fingers, toes, and mouth area. The nurse should suspect hypotension, frequent swallowing, feelings of fullness at the in-
which complication? cision site, choking, and bleeding. Thyroid storm is another term
for severe hyperthyroidism — not a complication of thyroidectomy.
Laryngeal nerve damage may occur postoperatively, but its signs
include a hoarse voice and, possibly, acute airway obstruction.
hyperparathyroidism

An elderly female client has been complaining of sleeping more, Hyperparathyroidism is most common in older women and is
increased urination, anorexia, weakness, irritability, depression, characterized by bone pain and weakness from excess parathy-
and bone pain that interferes with her going outdoors. Based roid hormone. Clients also exhibit hypercalciuria-causing polyuria.
on these assessment findings, the nurse should suspect which Although clients with diabetes mellitus and diabetes insipidus
disorder? have polyuria, they don't have bone pain and increased sleeping.
Hypoparathyroidism is characterized by urinary frequency rather
than by polyuria.
continue to monitor the client's blood glucose values.

The nurse should continue to monitor glucose in the blood to


prevent the client from continuing to experience hypoglycemia.
An elderly client who is receiving steroids has secondary diabetes One of the risk factors for hypoglycemia is decreased insulin clear-
and chronic kidney disease (CKD) and takes insulin. The client ance as with impaired kidney function and/or renal failure. Another
has had episodes of hypoglycemia. The nurse should: risk factor for hypoglycemia is increased glucose utilization when
there is too much activity or exercise without enough food.Protein
is digested slower than carbohydrate, but with chronic kidney
disease (CKD) it is more difficult for the kidneys to rid the body
of metabolic waste products.
It is planned around a wide variety of commonly available foods.

Each client should be given an individually devised diet selecting


The nurse is providing dietary teaching for a client with diabetes. commonly used foods from the Diabetic Association exchange
Which statement about the diet would be accurate? diet. Family members should be included in the diet teaching.
Nutritional requirements are not the same for all clients. Flexibility
is needed based on activity, not rigid control. Seasoning and
processed food should be managed.
Inform the HCP that the client has not received any subcutaneous
insulin yet.
A client who has been diagnosed with type 1 diabetes has an
ecause subcutaneous administration of insulin has a slower rate
insulin drip to aid in lowering the serum blood glucose level of 600
of absorption than IV insulin, there must be an adequate level of
mg/dL (33.3 mmol/L). The client is also receiving ciprofloxacin IV.
insulin in the bloodstream before discontinuing the insulin drip;
The health care provider (HCP) prescribes discontinuation of the
otherwise, the glucose level will rise. Adding an IV antibiotic has
insulin drip. What should the nurse do next?
no influence on the insulin drip; it should not be piggy-backed into
the insulin drip. Glargine cannot be administered IV and should
not be mixed with other insulins or solutions.
puffiness of the face and hands.

Hypothyroidism (myxedema) causes facial puffiness, extremity


A nurse should expect a client with hypothyroidism to report
edema, and weight gain. Signs and symptoms of hyperthyroidism
(Graves' disease) include an increased appetite, weight loss, ner-
vousness, tremors, and thyroid gland enlargement (goiter).

21 / 27
Unit of 3: Endocrine & Metabolic Disorders
Study online at https://quizlet.com/_8igkej
Review the one-time set-up for each new pen.
Inject in the thigh, abdomen, or upper arm.
Administer the drug within 60 minutes before morning and evening
meals.

Client teaching includes reviewing proper use and storage of


the exenatide dosage pen, particularly the one-time set-up for
A client is prescribed exenatide. What should the nurse instruct
each new pen. The nurse should instruct the client to inject the
the client to do? Select all that apply.
drug in the thigh, abdomen, or upper arm. The drug should be
administered within 60 minutes of the morning and evening meals;
the client should not inject the drug after a meal. The nurse should
review steps for managing hypoglycemia, especially if the client
also takes a sulfonylurea or insulin. If a dose is missed, the client
should resume treatment as prescribed, with the next scheduled
dose.
an occurrence of the excess loss of fluid associated with osmotic
diuresis
A client with diabetic ketoacidosis (DKA) has asked the unlicensed
nursing assistant for another pitcher of water. It is the third such Due to the DKA and fluid shift, the client would present with the 3
request over the past 4 hours. The nurse would recognize this Ps: polyuria, polyphagia, and polydipsia. Fatigue and weakness
request as which manifestation? may be caused by muscle wasting from the catabolic state of
insulin deficiency. The other choices are part of the problem but
not the main manifestation of the disease process.
tachycardia

Graves' disease, the most common type of thyrotoxicosis, is a


The nurse is completing a health assessment of a 42-year-old fe-
state of hypermetabolism. The increased metabolic rate generates
male with suspected Graves' disease. When conducting a focused
heat and produces tachycardia and fine muscle tremors. Anorexia
assessment, what should the nurse should assess the client for?
is associated with hypothyroidism. Loss of weight, despite a good
appetite and adequate caloric intake, is a common feature of
hyperthyroidism. Cold skin is associated with hypothyroidism.
idiopathic atrophy of the adrenal gland."

Primary Addison's disease refers to a problem in the gland itself


that results from idiopathic atrophy of the glands. The process is
A client newly diagnosed with primary Addison's disease asks the believed to be autoimmune in nature. The most common causes of
nurse about the cause of the disease. What should the nurse tell primary adrenocortical insufficiency are autoimmune destruction
the client? (70%) and tuberculosis (20%). Insufficient secretion of GH causes
"The disease is caused by: dwarfism or growth delay. Hyposecretion of glucocorticoids, al-
dosterone, and androgens occur with Addison's disease. Pituitary
dysfunction can cause Addison's disease, but this is not a primary
disease process. Oversecretion of the adrenal medulla causes
pheochromocytoma.
lethargy.

Although many of the disease signs and symptoms are vague and
nonspecific, most clients experience lethargy and depression as
A client has been diagnosed with Addison's disease. The nurse
early symptoms. Other early signs and symptoms include mood
should plan with the client to manage which effect of the disease?
changes, emotional lability, irritability, weight loss, muscle weak-
ness, fatigue, nausea, and vomiting. Most clients experience a
loss of appetite. Muscles become weak, not spastic, because of
adrenocortical insufficiency.
hypokalemia

Sodium retention is typically accompanied by potassium deple-


tion. Hypertension, hypokalemia, edema, and heart failure may re-
The nurse should monitor the client with Cushing's disease for
sult from the hypersecretion of aldosterone. The client with Cush-
which finding?
ing's disease exhibits postprandial or persistent hyperglycemia.
Clients with Cushing's disease have hypernatremia, not hypona-
tremia. Bone resorption of calcium increases the urine calcium
level.
22 / 27
Unit of 3: Endocrine & Metabolic Disorders
Study online at https://quizlet.com/_8igkej
dizziness when raising the arms above the head
dysphagia
respiratory distres
A client is diagnosed with a goiter after traveling in a foreign
A goiter can result from inadequate dietary intake associated with
country for 3 months. During the trip, the client could not tolerate
changes in diet or malnutrition. It is caused by insufficient thyroid
food. Which signs and symptoms would the nurse expect to see
gland production and depletion of glandular iodine. Signs and
in this client? Select all that apply.
symptoms of a goiter include enlargement of the thyroid gland,
dizziness when raising the arms above the head, dysphagia, and
respiratory distress. Cardiomegaly and oliguria are not associated
with a goiter.
Remind the client not to smoke, drink caffeinated beverages, or
change positions suddenly.
The nurse is assigned to a client with pheochromocytoma. In
The UAP is able to gather information such as vital signs, client
providing nursing care for the client, which action should the nurse
reports, and make environmental changes, and report these to
delegate to the unlicensed assistive personnel (UAP)?
the nurse. Assessing, teaching, and identifying stressful situations
that may trigger a hypertensive crisis requires additional skill and
education that is appropriate to the scope of practice of the RN.
Tell the nurse that she'd like to start at the beginning to be on the
safe side.
A staff member says she is really busy and asks the charge nurse
to double-check a dose of insulin which she has drawn up. The
The charge nurse should observe the process from the beginning
nurse holds up a bottle of Lente insulin, but the charge nurse
and determine whether the nurse is following the five rights of
notices a bottle of Lantus insulin on the medication cart. This
drug administration. Only then should she cosign that the dose is
nurse has made multiple medication errors and the charge nurse
correct. Saying that she can't check the dose unless she sees the
is concerned that she isn't safe. What should the charge nurse
nurse draw it up, asking the nurse which bottle of insulin she used,
do?
and asking to see the original order provide too much opportunity
for error.
client, nurse, pharmacist, and health care provider so the client
can participate in planning care with the entire team.

Learning goals are most likely to be attained when they are estab-
The nurse is teaching the client to self-administer insulin. Which lished mutually by the client and members of the health care team,
approach to establishing learning goals will likely be most effec- including the nurse, pharmacist, and health care provider.Learn-
tive? When the goals are established by the: ing is motivated by perceived problems or goals arising from
unmet needs. The perception of the unmet needs must be the
client's; however, the nurse, pharmacist, and health care provider
help the client arrive at his or her own perception of the need or
reason to learn.
increased urine osmolarity

In hyperglycemia, urine osmolarity (the measurement of dissolved


For a client with hyperglycemia, which assessment finding best particles in the urine) increases as glucose particles move into the
supports a nursing diagnosis of Deficient fluid volume? urine. The client experiences glucosuria and polyuria, losing body
fluids and experiencing deficient fluid volume. Cool, clammy skin;
jugular vein distention; and a decreased serum sodium level are
signs of fluid volume excess, the opposite imbalance.
suggest referral to a sex counselor or other appropriate profes-
A nurse is assigned to care for a postoperative client with diabetes sional.
mellitus. During the assessment interview, the client reports that
he's impotent and says he's concerned about the effect on his The nurse should refer this client to a sex counselor or other
marriage. In planning this client's care, the most appropriate in- professional. Making appropriate referrals is a valid part of plan-
tervention would be to ning the client's care. The nurse doesn't normally provide sex
counseling.
serum potassium level of 6.8 mEq/L (6.8 mmol/L)

A serum potassium level of 6.8 mEq/L indicates hyperkalemia,


which can occur in adrenal insufficiency as a result of reduced al-
dosterone secretion. A BUN level of 2.3 mg/dl is lower than normal.
23 / 27
Unit of 3: Endocrine & Metabolic Disorders
Study online at https://quizlet.com/_8igkej
A client in addisonian crisis is likely to have an increased BUN level
because the glomerular filtration rate is reduced. A serum sodium
level of 156 mEq/L indicates hypernatremia. Hyponatremia is more
A nurse is caring for a client in acute addisonian crisis. Which test
likely in this client because of reduced aldosterone secretion. A
result does the nurse expect to see?
serum glucose level of 236 mg/dl indicates hyperglycemia. This
client is likely to have hypoglycemia caused by reduced cortisol
secretion, which impairs glyconeogenesis.
hypercalcemia

Which condition should a nurse expect to find in a client diagnosed Hypercalcemia is the hallmark of excess parathyroid hormone
with hyperparathyroidism? levels. Serum phosphate will be low (hyperphosphatemia), and
there will be increased urinary phosphate (hyperphosphaturia)
because phosphate excretion is increased.
Take the insulin at around the same time each day at a meal.

The nurse should instruct the client to administer the insulin


around a meal because the onset of rapid acting insulin is im-
A nurse is teaching a client with diabetes mellitus about self-man-
mediate. The use of alcohol may cause hyperglycemia or hy-
agement. Which statement would be correct about the adminis-
poglycemia. The client should be instructed to monitor glucose
tration of lispro insulin?
level closely and should be discouraged from concurrent use.
Long-acting insulin is often taken in the evening once per day
because the insulin mimics the basal insulin secretion for a full
day. Lispro insulin can only be mixed with NPH insulin.
Administer 1 mg of dexamethasone orally at night and obtain
serum cortisol levels the next morning.

When Cushing's syndrome is suspected, a 24-hour urine collec-


A client diagnosed with Cushing's syndrome is admitted to the tion for free cortisol is performed. Levels of 50 to 100 mcg/day
hospital and scheduled for a dexamethasone suppression test. (1,379 to 2,756 nmol/L) in adults indicate Cushing's syndrome.
What should the nurse do during this test? If these results are borderline a high-dose dexamethasone sup-
pression test is done. The dexamethasone is given at 2300 to
suppress secretion of the corticotrophin-releasing hormone. A
plasma cortisol sample is drawn at 0800. Normal cortisol level less
than 5 mcg/dL (140 nmol/L) indicates normal adrenal response.
The isophane (NPH) insulin is peaking.

Headache, sweating, tremor, pallor, and nervousness typically


result from hypoglycemia, an insulin reaction in which serum glu-
cose level drops below 70 mg/dl (3.88 mmol/L). Isophane (NPH)
A client with diabetes mellitus has a prescription for 5 units of
insulin typically peaks at 4-12 hours after administration. However,
U-100 regular insulin and 25 units of U-100 isophane (NPH)
hypoglycemia may occur 4 to 18 hours after administration of iso-
insulin to be taken before breakfast. At about 4:30 p.m. (1630),
phane (NPH) insulin suspension or insulin zinc suspension, both
the client experiences headache, sweating, tremor, pallor, and
of which are intermediate-acting insulins. Although hypoglycemia
nervousness. What is the most probable cause of these signs and
may occur at any time, it usually precedes meals. Hyperglycemia,
symptoms?
in which serum glucose level is above 180 mg/dl (10 mmol/L),
causes such early manifestations as fatigue, malaise, and drowsi-
ness. Intravenous insulin can cause an acute shift in potassium
levels leading to hypokalemia, but these signs and symptoms
would include muscle weakness and muscle cramps.
Assess what the client already knows, then identify learning
needs.

It is most important to assess the client's teaching learning needs.


The nurse is caring for a client with type 2 diabetes who has The client needs to share what is already known about diabetes
been admitted with hyperglycemia. What is the most important and how it has been managed. Then it is important to identify the
consideration when developing a teaching plan for this client? client's level of motivation and what information is still needed. All
the listed information related to diabetes is important. However,
to individualize the teaching the nurse will need to assess what
the client already knows and direct the teaching to what is not
understood.

24 / 27
Unit of 3: Endocrine & Metabolic Disorders
Study online at https://quizlet.com/_8igkej
The skin flap appears white.

A white skin flap indicates lack of perfusion and the healthcare


Which assessment in a client that has just returned from having
provider should be notified immediately. Hoarseness may be due
a modified radical neck dissection with skin flap would require a
to trauma from the endotracheal tube that is inserted during
nurse to take immediate action?
surgery. Sutures may be visible after this surgery. An absence of
bowel sounds is a normal finding immediately post surgery with
general anesthesia.
Administer the insulin as ordered.

The nurse knows that a normal fasting blood sugar is between 72


A nurse obtained a client's fasting blood sugar (FBS) at 0700,
and 108 mg/dL (4 and 6 mmol/L). The result of 144 mg/dL indicates
which was 144 mg/dL (8 mmol/L). The client has an order for
that the client requires insulin to lower the blood glucose level. The
regular insulin 8 units every morning. What should the nurse do
nurse would not hold the insulin dose. Because there is alreadt a
next?
prescription for insulin, it is not necessary to contact the healthcare
provider at this time. Based on the FBS result, the nurse would
administer insulin before offering the client food.
preventing irreversible shock

Addison's disease is caused by a deficiency of adrenal corticos-


teroids and can result in severe hypotension and shock because
of uncontrolled loss of sodium in the urine and impaired mineralo-
corticoid function. This results in loss of extracellular fluid and dan-
Which goal is the priority for a client in Addisonian crisis?
gerously low blood volume. Glucocorticoids must be administered
to reverse hypotension. Preventing infection is not an appropriate
goal of care in this life-threatening situation. Relieving anxiety is
appropriate when the client's condition is stabilized, but the calm,
competent demeanor of the emergency department staff will be
initially reassuring.
methylprednisolone sodium succinate intravenously

A glucocorticoid preparation will be administered intravenously or


The nurse is caring for a client who is scheduled for an adrenalec- intramuscularly in the immediate preoperative period to a client
tomy. Which drug may be included in the preoperative prescrip- scheduled for an adrenalectomy. Methylprednisolone sodium suc-
tions to prevent Addison's crisis following surgery? cinate protects the client from developing acute adrenal insuffi-
ciency (Addison's crisis) that occurs as a result of the adrenalec-
tomy. Spironolactone is a potassium-sparing diuretic. Prednisone
is an oral corticosteroid. Fludrocortisones is a mineral corticoid.
Stop using iodized salt or iodized salt substitutes 1 week before
the scan.
Stop eating seafood 1 week before the scan.
Do not take any prescribed thyroid medication on the day of the
scan.

A thyroid scan visualizes the distribution of radioactive dye in the


A client visiting the clinic is scheduled for an outpatient thyroid
thyroid gland. Interventions before the scan include stopping the
scan in 2 weeks. Which instructions should the nurse include to
ingestion of iodine, which is found in iodized salt, salt substitutes,
ensure that this client is prepared for the test? Select all that apply.
and seafood. The client should also be instructed not to take
thyroid medication because it may interfere with the scan. The
client does not have to refrain from consuming food or fluids after
midnight if the scan is done on an outpatient basis. The radioactive
dye is administered intravenously. Routinely prescribed medica-
tions can be taken after the scan. Bed rest is maintained with a
thyroid biopsy, not a scan.
compensated respiratory alkalosis

The question states that the client has a history of acid-base


During shift report, the nurse learns the following laboratory val-
disturbance. The nurse would first note that the pH has returned to
ues: pH, 7.44; PCO2, 30mmHg; and HCO3,21 mEq/L for a client
close to normal indicating compensation. The nurse then assess
the PCO2 (normal: 35 to 45 mm Hg) and HCO3 (normal: 22 to
27mEq/L) levels. In a respiratory condition, the pH and the PCO2
25 / 27
Unit of 3: Endocrine & Metabolic Disorders
Study online at https://quizlet.com/_8igkej
move in opposite direction; thus, the pH rises and the PCO2 drops
(alkalosis) or vice versa (acidosis). In a metabolic condition, the pH
and the bicarbonate move in the same direction; if the pH is low,
with noted acid-base disturbances. Which acid-base imbalance is
the bicarbonate level will be low, also. In this client, the pH is at the
the client most likely experiencing?
high end of normal, indicating compensation and alkalosis. The
PCO2 is low, indicating a respiratory condition (opposite direction
of the pH).
"I will need to monitor blood glucose levels multiple times a day
while on the insulin pump."
A client is going to receive an insulin pump prior to discharge and
Based on the infusion of rapid-acting insulin, regular blood glucose
the nurse has done extensive teaching. Which statement indicates
monitoring is needed based on nutritional consumption and level
that the client has a good understanding about the pump?
of activity. Only rapid-acting insulin is used in pumps. Clients are
taught to bolus based on nutrition and consumption of diet. The
pump increases flexibility, not decreases.
fever
tachycardia
elevated liver enzymes
A nurse is managing the care of a client 10 days after a liver trans-
plant. What assessments may indicate organ rejection? Select all Transplant rejection is a Type IV hypersensitivity cell-mediated
that apply. immune response. Elevated temperature, tachycardia, and elevat-
ed liver enzymes are signs of liver transplant rejection. Because
the rejected liver is not processing bilirubin, the urine will be tea
colored and stool will be clay colored with rejection.
"Are you worried that you'll have the same experience as your
parent?"

Asking if the client feels they will have the same experience as
A client concerned about being diagnosed with type 2 diabetes
their parent gives the client an opportunity to vent underlying anx-
tells a nurse, "My parent suffered with diabetes for many years
iety. There's nothing to indicate that the client's parent's diabetes
and finally died of kidney failure in spite of treatment. Why should
wasn't under good control or that the parent had substandard care.
I try if I'm going to go through the same thing?" What is the nurse's
Saying there's no guarantee about how diabetes will progress
most appropriate response?
doesn't appropriately address the client's concerns and may in-
crease their anxiety. After the nurse has addressed the client's
anxiety, the nurse can more easily address more-specific teaching
needs.
nitroprusside

Excess catecholamine release occurs with pheochromocytoma


and causes hypertension. The nurse should prepare to administer
A nursing coordinator calls the intensive care unit (ICU) to inform nitroprusside to control the hypertension until the client undergoes
the department that a client with a suspected pheochromocytoma adrenalectomy to remove the tumor. Dopamine is used to treat
will be admitted from the emergency department. The ICU nurse hypotension, which is not associated with pheochromocytoma.
should prepare to administer which drug to the client? Pheochromocytoma does not affect blood glucose levels, so in-
sulin is not indicated in this client unless there is an underly-
ing diagnosis of diabetes mellitus. Lidocaine is sometimes used
to treat ventricular arrhythmias, which are not associated with
pheochromocytoma.
reduce the vascularity of the thyroid gland.

A client with a large goiter is scheduled for a subtotal thyroidecto- SSKI is frequently administered before a thyroidectomy because
my to treat thyrotoxicosis. Saturated solution of potassium iodide it helps decrease the vascularity of the thyroid gland. A highly
(SSKI) is prescribed preoperatively for the client. What should the vascular thyroid gland is very friable, a condition that presents
nurse explain to the client about the expected outcome of using a hazard during surgery. Preparation of the client for surgery
this drug? includes depleting the gland of thyroid hormone and decreasing
The drug helps: vascularity. SSKI does not decrease the progression of exophthal-
mos, and it does not decrease the body's ability to store thyroxine
or increase the body's ability to excrete thyroxine.

the levothyroxine before breakfast and the other medications 4


hours later.
26 / 27
Unit of 3: Endocrine & Metabolic Disorders
Study online at https://quizlet.com/_8igkej
The nurse is instructing the client with hypothyroidism who takes
Levothyroxine) must be given at the same time each day on an
levothyroxine 100 mcg, digoxin, and simvastatin. The nurse judges
empty stomach, preferably ½ to 1 hour before breakfast. Other
that the teaching regarding the use of these medications is effec-
medications may impair the action of levothyroxine absorption; the
tive if the client will take:
client should separate doses of other medications by 4 to 5 hours.
Initiate seizure precautions.

SIADH causes the release of excessive ADH resulting in flu-


A client with syndrome of inappropriate antidiuretic hormone id retention and dilutional hyponatremia. The client is exhibiting
(SIADH) is experiencing lethargy, weakness, headache, and mus- symptoms of hyponatremia, which can lead to seizures. Thus
cle aches. Which intervention is the nurse's priority? the priority is to place client on seizure precautions. Although
administering declomycin, increasing salt intake, and monitoring
serum osmolarity are appropriate interventions, they are not the
priority.
fresh fruits.

Cushing's syndrome causes sodium retention, which increases


When teaching a client with Cushing's syndrome about dietary urinary potassium loss. Therefore, the nurse should advise the
changes, the nurse should instruct the client to increase intake of client to increase intake of potassium-rich foods, such as fresh
fruit. The client should restrict consumption of dairy products,
processed meats, cereals, and grains because they contain sig-
nificant amounts of sodium.
glucocorticoids and androgens

The adrenal cortex is responsible for producing which sub- The adrenal glands have two divisions, the cortex and medulla.
stances? The cortex produces three types of hormones: glucocorticoids,
mineralocorticoids, and androgens. The medulla produces cate-
cholamines — epinephrine and norepinephrine.
Low serum calcium level stimulates parathyroid gland.
Parathyroid gland releases PTH.
Calcium is reabsorbed.
A nurse is caring for a client with a low calcium level. Place the High serum calcium level inhibits PTH secretion.
following options in chronological order to indicate the regulatory
feedback mechanism of parathyroid hormone (PTH) release in Simple feedback occurs when the level of one substance regulates
relation to calcium levels. All options must be used. the secretion of hormones. A low calcium level stimulates the
parathyroid gland to release PTH, which promotes resorption of
calcium, resulting in normalized calcium levels. When calcium
levels are elevated, PTH secretion is inhibited.
myxedema coma

A client receiving thyroid replacement therapy develops influenza Myxedema coma (severe hypothyroidism) is a life-threatening
and forgets to take the prescribed thyroid replacement medicine. condition that may develop if thyroid replacement medication isn't
The nurse understands that skipping this medication puts the taken. Although thyroid storm is life-threatening, it is caused by
client at risk for developing what life-threatening complication? severe hyperthyroidism. Systolic hypertension is associated with
thyroid storm. A cerebrovascular accident is not typically associ-
ated with hypothyroidism.

27 / 27
Passpoint - Endocrine and Metabolic Disorders
Study online at https://quizlet.com/_6qe5cu
A client with type 1 diabetes has a prescription for 5 units of
U-100 regular insulin and 25 units of U-100 isophane insulin sus-
pension to be taken before breakfast. At about 4:30 p.m. (1630),
Hypoglycemia
the client experiences headache, sweating, tremor, pallor, and
nervousness. What is the most probable cause of these signs and
symptoms?
A client has a serum calcium level of 7.2 mg/dl. During the physical
Trousseau's sign.
examination, the nurse expects to assess:
The client is being evaluated for hypothyroidism. The nurse should
decreased body temperature and cold intolerance.
stay alert for:
A client with primary diabetes insipidus is prescribed desmo-
"You may not be able to use desmopressin nasally if you have
pressin. Which instruction should the nurse provide before the
nasal discharge or blockage."
client is discharged?
When teaching a client about insulin administration, the nurse "Draw up clear insulin first when mixing two types of insulin in one
should include which instruction? syringe."
The nursing staff has just been trained how to use and care for a
new blood glucose monitor. Which nursing intervention demon- Calibrate the machine after installing a new battery.
strates proper use of a blood glucose monitor?
The nurse is teaching the client about risk factors for diabetes
Advanced age
mellitus. Which risk factor for diabetes mellitus is nonmodifiable?
The nurse is developing a teaching plan for a client diagnosed with
diabetes insipidus. The nurse should include information about Antidiuretic hormone (ADH)
which hormone lacking in clients with diabetes insipidus?
A client with type 2 diabetes hasn't received insulin coverage
for his afternoon blood glucose levels for 2 days. After further
investigation, a nurse discovers that the afternoon blood glucose
Notify the physician and complete an incident report.
levels were phoned in from the laboratory but weren't documented
in the client's medical record. What should the nurse do with this
information?
A nurse administers bromocriptine to a client diagnosed with
acromegaly. After administering the medication, the nurse realizes Verifying the client's identity on the identification band and med-
that she gave the medication to the wrong client. What could have ication administration record before providing the medication
been done to prevent this error?
A client presents with diaphoresis, palpitations, jitters, and tachy-
cardia approximately 4 hours after taking the prescribed usual Check blood glucose level, and administer carbohydrates.
morning insulin. What is the nurse's priority action?
The nurse is caring for a client with hypothyroidism. Which client
fatigue, cold intolerance, weight gain, and constipation
data would the nurse expect to collect?
After reinforcing education to a client on how to correctly self-ad-
"After taking my insulin out of the refrigerator, I'll draw up the clear
minister daily maintenance dose of 3 units of regular insulin and
insulin first to the line for 3 units and then cloudy insulin until there's
4 units of NPH insulin, which client statement demonstrates that
a total of 7 units in the syringe."
the education has been successful?
A client with hyperthyroidism develops high fever, extreme tachy-
cardia, and altered mental status. Which condition does the nurse thyroid storm
suspect is developing?
A client has been admitted after reporting acute abdominal pain in
the midepigastric region, back tenderness, nausea, and vomiting.
acute pancreatitis
The nurse recognizes these findings to be associated with which
condition?
The nurse is reinforcing education with parents of a child with
growth hormone deficiency. What sport should the nurse encour- gymnastics
age?
lethargy
When collecting data on an infant, which condition would alert the
nurse to a subtle sign of hypothyroidism? Select all that apply.
poor feeding

1/3
Passpoint - Endocrine and Metabolic Disorders
Study online at https://quizlet.com/_6qe5cu
A client has received dietary instructions as part of the treatment
"I can eat whatever I want as long as I cover the calories with
plan for diabetes type 1. Which statement by the client should alert
sufficient insulin."
the nurse that the client needs additional instructions?
The nurse is caring for a client with hypothyroidism. For which
levothyroxine
medication will the nurse reinforce instructions?
The nurse is reviewing laboratory data for a child with diabetes
insipidus. What characteristics of the urine would the nurse antic- pale; specific gravity less than 1.006
ipate observing?
The physician diagnoses type 1 diabetes in a client who has
classic manifestations of the disease and a random blood glucose
level of 350 mg/dl. In addition to dietary modifications, the physi-
human insulin.
cian prescribes insulin. Initially, most clients receive the least anti-
genic form of insulin. Therefore, the nurse expects the physician
to prescribe:
The nurse is teaching a client with type 1 diabetes how to treat
adverse reactions to insulin. To reverse a hypoglycemic reaction,
the client ideally should ingest an oral carbohydrate. However,
Glucagon
this treatment isn't always possible or safe. Therefore, the nurse
should advise the client to keep which alternate treatment on
hand?
The nurse is explaining the action of insulin to a client newly
diagnosed with diabetes mellitus. During the teaching, the nurse
beta cells of the pancreas.
reviews the process of insulin secretion in the body. The nurse is
correct when stating that insulin is secreted from the:
Early this morning, a client had a subtotal thyroidectomy. During
evening rounds, the nurse obtains data from the client, who now
has nausea, a temperature of 105° F (40.5° C), tachycardia, and Thyroid crisis
extreme restlessness. What is the most likely cause of these
signs?
The nurse is collecting data on a client with possible Cushing's
syndrome. In a client with Cushing's syndrome, the nurse would deposits of adipose tissue in the trunk and dorsocervical area.
expect to find:
A nurse is evaluating a client with hyperthyroidism. Which findings
weight loss, nervousness, and tachycardia
should the nurse anticipate that correlate with the diagnosis?
A client with type 1 diabetes has a highly elevated glycosylated
hemoglobin (Hb) test result. In discussing the result with the client, "It tells us about your sugar control for the last 3 months."
the nurse would be most accurate in stating:
A client with hyperparathyroidism declines surgery and is to re-
ceive hormone replacement therapy with estrogen and proges-
Maintain a moderate exercise program.
terone. Which instruction would be most important to include in
the client's teaching plan?
A female client who weighs 210 lb (95 kg) and has been diagnosed
with hyperglycemia tells the nurse that her husband sleeps in
another room because her snoring keeps him awake. The nurse
Acromegaly
notices that she has large hands and a hoarse voice. Which of
the following would the nurse suspect as a possible cause of the
client's hyperglycemia?
While administering morning medications, a nurse enters the
room of a client who recently had a thyroidectomy. She observes
Call for help.
that the client is sitting up in bed but appears unresponsive. After
confirming unresponsiveness, what should the nurse do next?
A client with type 2 diabetes was diagnosed with retinopathy. While
a nurse reviews the client's medication dosage, the client states, "I
Teach the client how to tell the difference between the medicine
can't read the names on the medicine bottles, so I hope I'm taking
bottles.
the right pills at the right time." What should the nurse do with this
information?

2/3
Passpoint - Endocrine and Metabolic Disorders
Study online at https://quizlet.com/_6qe5cu
A health care provider prescribes diet, exercise, and oral antidia-
betic agents for a client with diabetes. Which type of diabetes will type 2 diabetes
the nurse reinforce educating the client about?
A client with diabetes insipidus has had limited fluid intake over the
past 12 hours. For which complications should the nurse monitor severe dehydration and hypernatremia
the client?
The nurse is caring for a client who developed ketoacidosis. Which
insulin and IV fluids
prescribed treatment does the nurse anticipate administering?
A client is diagnosed with pituitary gigantism. The nurse reviews
growth hormone (GH)
the laboratory findings of which hormone value?
After undergoing a thyroidectomy, a client develops hypocalcemia
and tetany. Which medication should the nurse anticipate admin- calcium gluconate
istering?
A client with a family history of diabetes asks the nurse which
measures can be practiced to decrease the chance of devel-
"Start a moderate exercise program."
oping the disease. Which statement would be the nurse's best
response?
The nurse is gathering data from a child with juvenile hypothy- dry skin
roidism. Which common clinical finding would the nurse most likely
observe? Select all that apply. fatigue
The nurse is caring for a diabetic adolescent who admits to con-
suming many simple sugars and carbohydrates at a graduation
party. The parents brought the client to the emergency room with
unusual behavior. The serum glucose level was 375 mg/dL. The
health care provider provided a coverage schedule:

150 to 200 mg/dL—2 units of Humulin R.


201 to 250 mg/dL—4 units of Humulin R. 10 units
251 to 300 mg/dL—6 units of Humulin R.
301 to 350 mg/dL—8 units of Humulin R.
351 to 399 mg/dL—10 units of Humulin R.
Over 400 mg/dL—call the health care provider.

Mark the amount of insulin the nurse should draw into the
low-dose insulin syringe.
The nurse reinforces disease management instructions for a client
"Checking my blood sugar before meals and at bedtime will help
newly diagnosed with type 1 diabetes. Which statement indicates
me manage my blood sugar."
to the nurse that the client has understood the information?

3/3
Endocrine and Metabolic Disorders
Study online at https://quizlet.com/_3vofvo
While reviewing the day's charts, a nurse who's been under a
great deal of personal stress realizes that she forgot to administer
Report the error, complete the proper paperwork, and meet with
insulin to client with diabetes mellitus. She's made numerous
the unit manager
errors in the past few weeks and is now afraid her job is in jeopardy.
What is her best course of action?
Which of the following client statements during a health history
would indicate to the nurse that immediate further investigation is "No matter how much I drink, I am still thirsty all the time."
warranted?
An adolescent is to receive radioactive iodine for Graves' disease. "The advantage of radioactive iodine is that I will not need future
Which statement by the client reflects the need for more teaching? medication for my disease."
A nurse is caring for a client who was recently diagnosed with
hyperparathyroidism. Which statement by the client indicates the "I will increase my fluid and calcium intake."
need for additional discharge teaching?
A client is admitted to the health care facility for evaluation for
Addison's disease. Which laboratory test result best supports a Serum potassium level of 5.8 mEq/L (5.8 mmol/L)
diagnosis of Addison's disease?
Laboratory studies indicate a client's blood glucose level is 185
mg/dl (10.2 mmol/L). Two hours have passed since the client ate
Serum glycosylated hemoglobin (Hb A1c)
breakfast. Which test would yield the most conclusive diagnostic
information about the client's glucose use?
Which findings indicate that a client has developed water intoxi-
Confusion and seizures
cation secondary to treatment for diabetes insipidus
A client is scheduled for a transsphenoidal hypophysectomy to
remove a pituitary tumor. Preoperatively, the nurse should assess performing capillary glucose testing every 4 hours.
for potential complications by:
The nurse is assessing a client who has been admitted with im-
Absence of dorsalis pedis pulse, coolness, and decreased sen-
paired arterial circulation in the lower extremities due to diabetes
sation in the feet
mellitus. Which of the following would be expected findings?
A nurse is caring for a female client with hypothyroidism. The client
is extremely upset about her altered physical appearance. She Tell the client she'll soon experience improvement in her looks as
doesn't want to take her medication because she doesn't believe the medication corrects her hormone deficiency.
it's doing any good. What should the nurse do?
A client recently diagnosed with hyperparathyroidism demands to
"I'll get the chart and set up a time for you to review it with your
see what the physician has written about him in the chart. What is
physician."
the nurse's best response?
systolic blood pressure, 145 mm Hg
Which finding should the nurse report to the client's health care diastolic blood pressure, 87 mm Hg
provider (HCP) for a client with unstable type 1 diabetes mellitus? high-density lipoprotein (HDL), 30 mg/dL (1.7 mmol/L)
Select all that apply glycosylated hemoglobin (HbA1c), 10.2% (0.1)
triglycerides, 425 mg/dL (23.6 mmol/L
A nurse is teaching a client with type 1 diabetes how to treat
adverse reactions to insulin. To reverse hypoglycemia, the client
ideally should ingest an oral carbohydrate. However, this treat- Glucagon
ment isn't always possible or safe. Therefore, the nurse should
advise the client to keep which alternate treatment on hand?
A client with type 1 diabetes is admitted to an acute care facility
with diabetic ketoacidosis. To correct this acute diabetic emer- Initiate fluid replacement therapy
gency, which measure should the health care team take first?
During a follow-up visit to the physician, a client with hyper-
parathyroidism asks the nurse to explain the physiology of the
parathyroid glands. The nurse states that these glands produce phosphorus.
parathyroid hormone (PTH). PTH maintains the balance between
calcium and:
A woman with a progressively enlarging neck comes into the
clinic. She mentions that she has been in a foreign country for
the previous 3 months and that she didn't eat much while she
1/8
Endocrine and Metabolic Disorders
Study online at https://quizlet.com/_3vofvo
was there because she didn't like the food. She also mentions that
she becomes dizzy when lifting her arms to do normal household
Goiter
chores or when dressing. What endocrine disorder should the
nurse expect the physician to diagnose?
The emergency department (ED) nurse is caring for a client with
a possible acid-base imbalance. The physician has ordered an
Bicarbonate
arterial blood gas (ABG). What is one of the most important
indications of an acid-base imbalance that is shown in an ABG?
A client has been admitted to the hospital for the treatment of
diabetic ketoacidosis, a problem that was accompanied by a ran-
dom blood glucose reading of 31.9 mmol/L (575 mg/dL), vomiting, Exacerbation
and shortness of breath. This client has experienced which of the
following phenomena?
"I need to make sure that I eat my meals and snacks on time after
I take my insulin."
The nurse is teaching a client with type I diabetes self-administra-
"If I monitor and control my blood glucose levels carefully, there
tion of insulin. Which statement by the client would be an expected
is less likelihood of suffering long-term complications."
outcome of the teaching session? Select all that apply.
"If I exercise more than is normal, there is a risk that I might
become hypoglycemic."
After a 3-month trial of dietary therapy, a client with type 2 diabetes
still has blood glucose levels above 180 mg/dl (9.99mmol/L).
at breakfast
The physician adds glyburide, 2.5 mg P.O. daily, to the treatment
regimen. The nurse should instruct the client to take the glyburide:
When conducting a health history with a female client with thy-
rotoxicosis, the nurse should ask about which change in the oligomenorrhea
menstrual cycle?
Pancreatic enzyme replacements are prescribed for the client with
chronic pancreatitis. When should the nurse instruct the client to with each meal and snack
take them to obtain the most therapeutic effect?
The nurse is assessing a client with hepatitis A and notices
that the aspartate transaminase (AST) and alanine transaminase
"I take acetaminophen for arthritis pain."
(ALT) lab values have increased. Which statement by the client
indicates the need for further instruction by the nurse?
A client with insulin-dependent diabetes develops a seizure disor-
der and has been prescribed phenytoin. Which information should report changes in blood glucose levels to the health care provider.
the nurse include in the teaching plan? The client should: (Select use a soft toothbrush and floss the teeth daily
all that apply.
A client is admitted for treatment of the syndrome of inappropriate
antidiuretic hormone (SIADH). Which nursing intervention is ap- Restricting fluids
propriate?
A physician orders an isotonic I.V. solution for a client. Which
Lactated Ringer's solution
solution should the nurse plan to administer?
Which intervention is the most critical for a client with myxedema
Maintaining a patent airway
coma?
A client with hyperparathyroidism declines surgery and is to re-
ceive hormone replacement therapy with estrogen and proges-
"Maintain a moderate exercise program."
terone. Which instruction is most important for the nurse to include
in the client's teaching plan?
A client with status asthmaticus requires endotracheal intubation
and mechanical ventilation. Twenty-four hours after intubation, the
client is started on the insulin infusion protocol. The nurse must Sweating, tremors, and tachycardia
monitor the client's blood glucose levels hourly and watch for
which early signs and symptoms associated with hypoglycemia?
The nurse is receiving results of a blood glucose level from the write down the results, read back the results to the caller from the
laboratory over the telephone. The nurse should: laboratory, and receive confirmation from the caller.

2/8
Endocrine and Metabolic Disorders
Study online at https://quizlet.com/_3vofvo
A group of nursing assistants hired for the medical-surgical floors
are attending hospital orientation. Which topic should the educa-
Obtaining, reporting, and documenting fingerstick glucose levels
tor cover when teaching the group about caring for clients with
diabetes mellitus?
The nurse should teach the diabetic client that which is most
nervousness
indicative of hypoglycemia?
An incoherent client with a history of hypothyroidism is brought
to the emergency department by the rescue squad. Physical and
laboratory findings reveal hypothermia, hypoventilation, respira-
tory acidosis, bradycardia, hypotension, and nonpitting edema of myxedema coma.
the face and pretibial area. Knowing that these findings suggest
severe hypothyroidism, the nurse prepares to take emergency
action to prevent the potential complication of:
A client is diagnosed with syndrome of inappropriate antidiuretic
hormone (SIADH). The nurse informs the client that the physician
will order diuretic therapy and restrict fluid and sodium intake to Cerebral edema
treat the disorder. If the client doesn't comply with the recom-
mended treatment, which complication may arise?
For a client with hyperthyroidism, treatment is most likely to in-
a thyroid hormone antagonist
clude:
Every morning, a client with type 1 diabetes receives 15 units of
70% NPH insulin and 30% regular insulin
Humulin 70/30. What does this type of insulin contain?
A client is placed on hypocalcemia precautions after removal of Numbness
the parathyroid gland for cancer. The nurse should observe the Tingling
client for which symptoms? Select all that apply. Muscle twitching and spasms
A client has been diagnosed with metabolic alkalosis. The nurse
should anticipate what finding from the client's arterial blood gas- Serum bicarbonate of 28 mEq/L
es?
The nurse is caring for a client in the medical unit. The nurse
receives a health care provider's order for Hydrocortisone 100 mg
intravenously at a rate of 10 cc/hour for a client in acute adrenal Addison's disease
crisis. The nurse is most correct to understand that this treatment
is common in clients with which disease process?
A client with primary diabetes insipidus is ready for discharge
"You may not be able to use desmopressin nasally if you have
on desmopressin (DDAVP). Which instruction should the nurse
nasal discharge or blockage."
provide?
Bone resorption is a possible complication of Cushing's disease.
To help the client prevent this complication, the nurse should maintain a regular program of weight-bearing exercise.
recommend that the client:
Because of steroid excess after a bilateral adrenalectomy, the
delayed wound healing
nurse should assess the client for:
When teaching a client when to take glipizide in order to maximize
the effectiveness of the drug, the nurse should instruct the client take glipizide 30 minutes before breakfast.
to:
During a class on exercise for clients with diabetes mellitus, a
client asks the nurse educator how often to exercise for thirty
At least five times per week
minutes. Which of the exercise frequency would meet the goals
of planned exercise?
An agitated, confused client arrives in the emergency depart-
ment. The client's history includes type 1 diabetes, hyperten-
sion, and angina pectoris. Assessment reveals pallor, diaphore-
sis, headache, and intense hunger. A stat blood glucose sample 15 g of a simple carbohydrate
measures 42 mg/dl, (2.3 mmol/L) and the client is treated for an
acute hypoglycemic reaction. After recovery, the nurse teaches
the client to treat hypoglycemia by ingesting:

3/8
Endocrine and Metabolic Disorders
Study online at https://quizlet.com/_3vofvo
A female client with hyperglycemia who weighs 210 lb (95 kg) tells
the nurse that her husband sleeps in another room because her
snoring keeps him awake. The nurse notices that the client has Acromegaly
large hands and a hoarse voice. Which disorder would the nurse
suspect as a possible cause of the client's hyperglycemia?
Two weeks after a partial thyroidectomy, a client is being seen
Hair loss.
for the postoperative follow-up appointment. The nurse is aware
Dry skin.
that the client is at increased risk for hypothyroidism. Which signs
Cold intolerance.
and symptoms would the nurse anticipate in a client with hypothy-
Fatigue.
roidism? Select all that apply.
Amount
When reviewing the urinalysis report of a client with newly di-
Odor
agnosed diabetes mellitus, the nurse would expect which urine
Glucose level
characteristics to be abnormal? Select all that apply.
Ketone bodies
When a client demonstrates the technique for self-administering
pulling back on the syringe plunger as soon as the needle is in
NPH insulin, which action indicates that the client needs additional
subcutaneous tissue
teaching?
A client is diagnosed with syndrome of inappropriate antidiuretic
hormone (SIADH). The nurse should assess the client for which decreased serum sodium level
alteration in fluid and electrolyte balance?
Which action is most effective when a nurse is assessing the client
measuring urine output hourly
suspected of developing diabetes insipidus
Which nursing diagnosis takes highest priority for a client with Imbalanced nutrition: Less than body requirements related to thy-
hyperthyroidism? roid hormone excess
A client with a tentative diagnosis of hyperosmolar hyperglycemic
nonketotic syndrome (HHNS) has a history of type 2 diabetes that
Serum osmolarity
is being controlled with an oral diabetic agent, tolbutamide. Which
laboratory test is the most important for confirming this disorder?
The nurse is evaluating a client with hyperthyroidism who is tak-
ing propylthiouracil (PTU) 100 mg/day in three divided doses for
"I am able to sleep and rest at night."
maintenance therapy. Which statement from the client indicates
the drug is effective?
The client with type 1 diabetes mellitus is taught to take isophane
insulin suspension NPH at 1700 each day. The client should be
0100, while sleeping.
instructed that the greatest risk of hypoglycemia will occur at about
what time?
A client with diabetes has been diagnosed with hypertension,
and the health care provider (HCP) has prescribed atenolol, a
an increase in the hypoglycemic effects of insulin
beta-blocker. When performing discharge teaching, it is important
for the nurse to emphasize that the addition of atenolol can cause:
Capillary glucose monitoring is being performed every 4 hours for
a client diagnosed with diabetic ketoacidosis. Insulin is adminis-
tered using a scale of regular insulin according to glucose results.
onset to be at 2:30 p.m. (1430) and its peak to be at 4 p.m.(1600).
At 2 p.m. (1400), the client has a capillary glucose level of 250
mg/dl for which he receives 8 units of regular insulin. The nurse
should expect the dose's:
The nurse is teaching a client about insulin administration. Which
statement if made by a client would indicate to the nurse the client "I will use my abdominal injection site if I want to jog."
understands insulin administration teaching?
A client is admitted with a diagnosis of diabetic ketoacidosis.
An insulin drip is initiated with 50 units of insulin in 100 ml of
normal saline solution administered via an infusion pump set at
5
10 ml/hour. The nurse determines that the client is receiving how
many units of insulin each hour? Record your answer using a
whole number.
A client who was diagnosed with type 1 diabetes 14 years ago
is admitted to the medical-surgical unit with abdominal pain. On
4/8
Endocrine and Metabolic Disorders
Study online at https://quizlet.com/_3vofvo
admission, the client's blood glucose level is 470 mg/dl (26.1
mmol/L). Which finding is most likely to accompany this blood Rapid, thready pulse
glucose level?
A nurse is planning care for a client in acute addisonian crisis.
Decreased cardiac output
Which nursing diagnosis should receive the highest priority?
A client diagnosed with hyperosmolar hyperglycemic nonketotic
syndrome (HHNS) is stabilized and prepared for discharge. When "I can avoid getting sick by not becoming dehydrated and by
preparing the client for discharge and home management, which paying attention to my need to urinate, drink, or eat more than
statement indicates that the client understands the condition and usual."
how to control it?
A client is ordered prednisone daily. Which statement best ex-
Morning administration of prednisone mimics the body's natural
plains why the nurse should instruct the client to take this drug in
corticosteroid secretion pattern
the morning?
The nurse is completing a health assessment of a 42-year-old
female with suspected Graves' disease. The nurse should assess tachycardia
this client for:
he client who has been hospitalized with pancreatitis does not
drink alcohol because of religious convictions. The client comes
upset when the health care provider (HCP) persists in asking there is a strong link between alcohol use and acute pancreatitis
about alcohol intake. The nurse should explain that the reason for
these questions is that:
The nurse is teaching the client to self-administer insulin. Learning client, nurse, pharmacist, and health care provider so the client
goals most likely will be attained when they are established by the: can participate in planning care with the entire team.
For a client with Graves' disease, which nursing intervention pro-
Maintaining room temperature in the low-normal range
motes comfort?
Upon shift report, the nurse learns the following laboratory values:
pH, 7.44; PCO2, 30mmHg; and HCO3,21 mEq/L for a client with
Compensated respiratory alkalosis
noted acid-base disturbances. Which acid-base imbalance is the
client most likely experiencing?
The nurse is admitting a client with newly diagnosed diabetes
mellitus and left-sided heart failure. Assessment reveals low blood
pressure, increased respiratory rate and depth, drowsiness, and
Metabolic acidosis.
confusion. The client reports headache and nausea. Based on the
serum laboratory results below, how would the nurse interpret the
client's acid-base balance?
On a medical-surgical floor, a nurse is caring for a cluster of clients
A 55-year-old complaining of chest pressure
with diabetes mellitus. Which client should the nurse assess first?
When preparing to draw up 8 units of a short-acting insulin and
20 units of a long-acting insulin in the same syringe, the nurse inject air in the vial with the long-acting insulin first.
should:
Which factor, if described by the parents of a child with cystic
fibrosis (CF), indicates that the parents understand the underlying an abnormality in the body's mucus-secreting glands
problem of the disease?
A nurse is teaching an 8-year-old with diabetes and her parents
about managing diabetes during illness. The nurse determines the
more insulin.
parents understand the instruction when they indicate that when
the child is ill they will provide:
The nurse teaches the client to report signs and symptoms of
hypopituitarism
which potential complication after hypophysectomy?
The nurse is instructing a college student with Addison's disease
how to adjust the dose of glucocorticoids. The nurse should ex-
having wisdom teeth extracted
plain that the client may need an increased dosage of glucocorti-
coids in which situation?
When teaching a client about taking oral glucocorticoids, how
with meals or with an antacid
should the nurse instruct the client to take the medication?

5/8
Endocrine and Metabolic Disorders
Study online at https://quizlet.com/_3vofvo
The client with Cushing's disease needs to modify dietary intake to
control symptoms. In addition to increasing protein, which strategy Restrict sodium.
would be most appropriate?
A client with hypothyroidism (myxedema) is receiving levothyrox-
ine, 25 mcg P.O. daily. Which finding should the nurse recognize Tachycardia
as an adverse reaction to the drug?
A nurse should perform which intervention for a client with Cush- Explain that the client's physical changes are a result of excessive
ing's syndrome? corticosteroids.
A client with a history of Addison's disease and flulike symptoms
accompanied by nausea and vomiting over the past week is
brought to the facility. His wife reports that he acted confused and
was extremely weak when he awoke that morning. The client's
Hydrocortisone
blood pressure is 90/58 mm Hg, his pulse is 116 beats/minute, and
his temperature is 101° F (38.3° C). A diagnosis of acute adrenal
insufficiency is made. What should the nurse expect to administer
by I.V. infusion?
The nurse is educating a client on diabetes management. The
client is asking questions that cause the nurse to be concerned
about the client's ability to retain the information. Which of the Repeat important information during the presentation.
following would be the best technique for the nurse to use to
enhance the retention of information by the client?
The nurse is caring for a client in a diabetic coma. The nurse is
aware that this is caused by an excess of which substance in the Ketones from rapid fat breakdown, causing acidosis
blood
A client is brought to the emergency department after wandering
on the street. The client is confused, verbalizes double vision,
"What have you eaten today?"
headache, and shakiness. Laboratory data reveal a serum blood
"Do you take insulin or oral antidiabetic medication?"
glucose of 52 mg/dL (52 mmol/L). Which questions, asked by
"Have you ever felt this way before?"
the nurse, may reveal more data related to the client's condition?
Select all that apply.
The nurse is caring for a client with possible Cushing's syndrome
undergoing diagnostic testing. The health care provider orders lab Cortisol levels before and after the system is challenged with a
work and a dexamethasone suppression test. Which parameter synthetic steroid
would the nurse assess on the dexamethasone suppression test?
A client with diabetes mellitus has a foot ulcer. The physician
orders bed rest, a wet-to-damp dressing change every shift, and They debride the wound and promote healing by secondary inten-
blood glucose monitoring before meals and at bedtime. Why are tion.
wet-to-damp dressings used for this client?
Which instruction should a nurse give to a client with diabetes
"Test your blood glucose every 4 hours."
mellitus when teaching about "sick day rules"?
A client whose physical findings suggest a hyperpituitary condition
undergoes an extensive diagnostic workup. Test results reveal a
pituitary tumor, which necessitates a transsphenoidal hypophy-
"You must avoid coughing, sneezing, and blowing your nose."
sectomy. The evening before the surgery, the nurse reviews pre-
operative and postoperative instructions given to the client earlier.
Which postoperative instruction should the nurse emphasize?
A client is diagnosed with syndrome of inappropriate antidiuretic
hormone (SIADH). The nurse should anticipate which laboratory Serum sodium level of 124 mEq/L
test result
A nurse is providing dietary instructions to a client with hypo-
consuming a low-carbohydrate, high-protein diet and avoiding
glycemia. To control hypoglycemic episodes, the nurse should
fasting.
recommend:
A nurse is caring for a client with diabetes insipidus. The nurse
vasopressin.
should anticipate administering:

A client tells the nurse that she has been working hard for the past
glycosylated hemoglobin level.
3 months to control her type 2 diabetes with diet and exercise. To

6/8
Endocrine and Metabolic Disorders
Study online at https://quizlet.com/_3vofvo
determine the effectiveness of the client's efforts, the nurse should
check:
Parathyroid hormone (PTH) has which effects on the kidney? Stimulation of calcium reabsorption and phosphate excretion
An elderly client with type 2 diabetes had hyperglycemic hyper-
osmolar syndrome (HHS). The nurse should monitor the infusion cerebral edema.
for too rapid correction of the blood glucose in order to prevent:
Which instruction about levothyroxine administration should a
"Take the drug on an empty stomach."
nurse teach a client?
During an emergency, a physician has asked for I.V. calcium to
Check with the physician for his complete order.
treat a client with hypocalcemia. The nurse should:
The nurse should teach the client with hepatitis A to: increase carbohydrates and protein in the diet.
The nurse should assess a client taking chlorpropamide for: hypoglycemia.
A nurse is teaching a client with diabetes mellitus about self-man-
agement. Which of the following would be correct about the ad- Take the insulin at around the same time each day at a meal.
ministration of lispro insulin?
Which condition should a nurse expect to find in a client diagnosed
Hypercalcemia
with hyperparathyroidism?
Which topic is most important to include in the teaching plan for a
client newly diagnosed with Addison's disease who will be taking The importance of watching for signs of hyperglycemia.
corticosteroids?
The physician has prescribed sodium chloride for a hospitalized Compare ABG findings with previous results.
51-year-old client in metabolic alkalosis. Which nursing actions Maintain intake and output records.
are required to manage this client? Select all that apply. Document presenting signs and symptoms.
Serum sodium level
The nurse is caring for a client with Cushing's disease. During
Serum potassium level
change of shift report, which assessment laboratory data would
Blood glucose level
the nurse anticipate communicating? Select all that apply.
White blood cell count
Propylthiouracil (PTU) is prescribed for a client with Graves' dis-
sore throat
ease. The nurse should teach the client to immediately report:
A male client expresses concern about how a hypophysectomy
will affect his sexual function. Which statement provides the most Removing the source of excess hormone should restore the
accurate information about the physiologic effects of hypophysec- client's libido, erectile function, and fertility.
tomy in a male?
The nurse should institute which measure to prevent transmission
decreasing contact with blood and blood-contaminated fluids
of the hepatitis C virus to health care personnel?
A client with a history of chronic hyperparathyroidism admits to
being noncompliant. Based on initial assessment findings, the
nurse formulates the nursing diagnosis of Risk for injury. To com- Related to bone demineralization resulting in pathologic fractures
plete the nursing diagnosis statement for this client, which "relat-
ed-to" phrase should the nurse add?
A nurse is assessing a client after a thyroidectomy. The assess-
ment reveals muscle twitching and tingling, along with numbness
tetany
in the fingers, toes, and mouth area. The nurse should suspect
which complication?
A client with a history of hypertension is diagnosed with primary
hyperaldosteronism. This diagnosis indicates that the client's hy-
Adrenal cortex
pertension is caused by excessive hormone secretion from which
gland?
A client is diagnosed with syndrome of inappropriate antidiuretic
hormone (SIADH). Laboratory results reveal serum sodium level
Restricting fluids to 800 ml/day
130 mEq/L and urine specific gravity 1.030. Which nursing inter-
vention helps prevent complications associated with SIADH?

"Ketones will tell us if your body is using other tissues for energy."

7/8
Endocrine and Metabolic Disorders
Study online at https://quizlet.com/_3vofvo
A client newly diagnosed with diabetes mellitus asks why he
needs ketone testing when the disease affects his blood glucose
levels. How should the nurse respond?
The nurse should teach the client with chronic pancreatitis to
monitor the effectiveness of pancreatic enzyme replacement ther- observing stools for steatorrhea.
apy by:
A 48-year-old female client is seen in the clinic for newly diag- High-fiber, low-calorie diet
nosed hypothyroidism. Which topics should the nurse include in a Use of stool softeners
client teaching plan? Select all that apply. Thyroid hormone replacements
A client is admitted to the hospital with signs and symptoms of Excessive thirst
diabetes mellitus. Which of the following findings is the nurse most Excessive hunger
likely to observe in this client? Select all that apply. Frequent, high-volume urination
The nurse is caring for a client on the urinary unit. When providing
report to the next shift, it is noted that the client has osteopenia
Hyperparathyroidism
and history of renal calculi. Which of the following disorders would
the nurse suspect?
The health care provider (HCP) has prescribed insulin detemir for
a client with type 2 diabetes requiring insulin. The nurse should "You do not mix insulin detemir; the solution is clear."
tell the client:

8/8
NCLEX- CH 61 ENDOCRINE AND METABOLIC DISORDERS
Study online at https://quizlet.com/_2f5t5u
Pt with diabetes requests med for headache soon after returning ANSWER: 2
from early morning x-ray. The nurse observes the pt is upset about
headache, angry at missing breakfast, and has moist hands. What Headache, restlessness, anxiety, sweating, and increased pulse
priority action should the nurse take at this time? are S/S of hypoglycemia. Resolution of symptoms should occur
after the pt drinks the juice.
1. administer medication for headache and arrange for a breakfast
tray Treating the headache and obtaining a breakfast tray fail to rec-
ognize the pt's actual problem. Acknowledging dissatisfaction,
2. check BG level and be prepared to give 4 oz of juice immediately obtaining a snack and giving meds address the pt's concerns but
do not verify the pt's BG as a possible etiology for the symptoms.
3. acknowledge his dissatisfaction, offer a snack, and give the med
Treating the headache and checking labs fails to address the
4. administer the headache med and review the days lab test immediate risk of hypoglycemia, which can be addressed by
results checking BG
A 70 yr pt admitted with BG of 750 is being TX for HHNK wit
IV regular insulin at 10u/hr, NS with 20mEq of K per L infusing ANSWER: 3
at 250mg/h, and O2 @ 2l/m. The pt has been oriented when
stimulated, and BG has dropped to 400. The pt now demands to Incerased preload caused by the IV infusion at 250ml/h may
get out of bed and his skin feels cool and moist. What should the exceed the myocardium's workload capacity, leading to signs of
nurse do at this time? decreased cardiac output and CHF.

1. Interpret this as a sign of hypoglycemia and ck his BG There is no risk for hypoglycemia while the BG is still elevated to
400.
2. Recognize the pt is feeling better and is seeking control of his
situation The nurse should seek a physiological basis for the chane in pt's
status rather than seeking control, esp since skin is cool and moist.
3. Auscultate breath sounds and assess O2 sat
Checking for bladder distention or fever represents a failure to
4. Assess the pt for bladder distention or signs of imbalanced body directly assess for signs of possible fluid overload.
temp
Pt who underwent a colonoscopy after a premedication with mi-
dazolam (Versed) returns to the nursing unit. The pt is given
morphine 2mg IV push for abd pain associated with BP 140/80,
pulse 78, RR 20. Twent min later, the pt is lethargic, has weak ANSWER: 4
hand grasps, weak peripheral pulse of 88, BP 120/66, RR 14. How
should the nurse interpret these assessment findings? Decreased LOC, weak hand grasp and peripheral pulses with
increased HR and decreased BP result from acidosis. These are
1. The pt is resting with pain relieved signs of respiratory acidosis secondary to hypoventilation from the
midazolam.
2. The pt is now showing signs of dehydration because of the colon
procedure prep Concluding the pt is resting and pain is relieved fails to address
the falling RR and change in LOC.
3. The pt is now fatigued because of anxiety, pain, and fear of
outcome of the procedure Fatigue would not account for the weak peripheral pulse.

4. The pt is experiencing impaired gas exchange because of


hypoventilation
ANSWER: 1
The pt is schedule for bilateral adrenalectomy as TX for an adrenal
cortex tumor. What is the nurse's highest priority for this pt in the During the 1st 48hr after adrenalectomy, pts are at risk for adrenal
IMMEDIATE post-op period? insufficiency and hypovolemic shock. The lack of corisol produc-
tion can cause fluid and electrolyte loss and hypoglyemia.
1. assess fluid and electrolyte balance, signs of hypoglycemia,
and hypotension Peripheral edema is more likely assoc with excess fluid volume
but the risk after adrenalectomy is deficient fluid volume caused
2. assess for the signs of hypoxia, cardiac arrhythymias, and by sudden decrease in circulating corticosteroids and mineralo-
peripheral edema corticoids.

3. monitor the incision integrity, peripheral pulses, and magnesium Incision integrity and peripheral pulses are routine assessments,
level and magnesium level is not a particular concern at this time.

Assessing for hyperthermia, bed mobility are part of routine


1/3
NCLEX- CH 61 ENDOCRINE AND METABOLIC DISORDERS
Study online at https://quizlet.com/_2f5t5u
4. assess for hyperthermia, bed mobility, pupil reaction, and eye post-op assessments, and neurological assessment of pupils and
movement eye movements is not particular concern after adrenalectomy
A female pt has been taking propylthiouracil (PTU) for 5 month to
TX hyperthyroidism. After falling and spraining her ankle, she is
treated and is given crutch walking instructions. She says she will
never have enough energy to get around on crutches and is upset
ANSWER: 1
about the 10lbs she gained this winter. What should be the nurse's
first action?
The pt reports of lack of energy and wt gain are consistent with
HYPOthyroidism, which is DX with a serum T4.
1. document the pt's statements and consult the dr to order a
serum T4
Considering the pt's complaints of energy deficit, the recent fall
causing the sprain, and info about the thyroid med the nurse is
2. Discharge the pt to home and encourage her to have a TSH
obligated to consult the Dr for T4 eval to prevent further injury.
level drawn
Discharging the pt ignores the pt's concern and a TSH level is
3. Encourage the pt to rest at home until the sprain is healed, then
expected to be high in HYPERthyroidism and may not be of use.
increase activity

4. investigate the availability of a walking splint instead of using


the crutches
ANSWER: 1

Blood on the gown indicates excessive incisional bleeding. Breath


The pt is 6hrs post thyroid surgery. The nursing assistant reports sounds, including auscultating over the tracheal area, and respi-
that the pt is upset because there is blood on the pt's gown. What ratory effort should be assessed FIRST to determine if edema is
is the PRIORITY action of the nurse? present in the tissues, thus compromising the airway.

1. assess the pt's breath sounds and respiratory effort After thoroughly assessing pt, the nurse should inform surgeon
amt of bleeding and all other assessment data.
2. state that it is normal to have some bleeding and ask the nurse
aide to change the gown Usually, w thyroid surgery there is min. bleeding post-op and
having the nurse's aide change the gown fails to provide proper
3. reassure the pt that some bleeding is normal, and then assess pt assessment.
the pt's level of pain
Focusing on pt's pain level doesnt address the bleeding, which is
4. reinforce the drsg, change the gown, and call the surgeon excessive rather than normal.

Reinforcing the drsg and calling the surgeon fails to address the
need for assessing the pt's airway
ANSWER: 4, 5
A pt who was admitted with HHNK asks how he can prevent
recurrence of this illness. The nurse would instruct the pt about HHNK is assoc with HYPERglycemic response to infection or
which helpful prevention measures? other disease or illness, some meds, dehydration, stress induced
SELECT ALL THAT APPLY hyperglycemia, or a combination of these factors.

1. use sliding-scale insulin to cover periodic snacks that are not The response to stress can increase blood glucose levels so
part of the dietary plan stress management may be helpful as part of overall measures
to prevent increased BG.
2. maintain fluid balance by drinking 4 glasses of water daily
HHNK occures in pts with TYPE 2, primarily older adults, and thus
3. monitor for signs of infection and treat infection early insulin is not part of the usual TX plan.

4. consult primary when fasting blood glucose is elevated Drinking 4 glasses of water daily is insufficient; 6-8 are recom-
mended for gen. health
5. use stress management techniques because the stress re-
sponse increases blood glucose Consulting a Dr for elevated BG does not demonstrate an under-
standing of how to prevent HHNK

A pt with HYPERparathyroidism is admitted with cardiac dysrhyth- ANSWER: 4


mias, including bursts of SVT AND PVCs. The pt asks why the
cardiologist prescribed so much IV fluid and then Lasix. What is HYPERparathyroidism causes hypercalcemia.
the best explanation?
2/3
NCLEX- CH 61 ENDOCRINE AND METABOLIC DISORDERS
Study online at https://quizlet.com/_2f5t5u
Large volume saline infusion given with lasix will stimulate the
1. improve cardiac output kidneys to excrete calcium.
2. eliminate metabolic wastes
3. replace missing electrolytes In acute situations requiring rapid reduction, pts could also be
4. promote excretion of calcium given IV calcitonin and phosphates.
ANSWER: 1,2,5

Pt with HYPOparathyroidism is to be D/C home after stabilization Pt should be taught to keep follow up apts for lab tests and with
of fluid and electrolyte levels. Which are critical concepts that the Dr consistently
nurse should teach the pt prior to D/C? SELECT ALL THAT APPLY
Pts with HYPOparathyroidism have paresthesia, muscle spasms,
1. importance of keeping follow up apts for lab test and with Dr and hyperactive reflexes which put them at risk for falling

2. Strategies that will help decrease the risk of falling Fall prevention measures are important for this pt

3. significant signs of hypoglycemia to monitor for and report Hypoglycemia is a primary concern in DM, a disorder of the
pancreas.
4. S/S of renal calculi to monitor for and report
Low serum calcium levels do not predispose the pt to renal calculi.
5. how to plan meals that include increased amts of calcium and
Vit D Pts with HYPOparathyroidism have low serum calcium levels and
require increased calcium in the diet, and often calcium cupple-
ments and increased Vit D
ANSWER: 2

A pt with SIADH is receiving continuous enteral nutrition. Con- Pts with SIADH are encouraged to drink fluids HIGH IN SODIUM,
sidering the impact of the disorder on fluid balance, what action so they should have feedings flushed with NS.
should the nurse take when working with the enteral feeding tube?
To prevent electrolyte loss, all of the residual that is aspirated from
1. discard the 50ml residual and replace it with 50ml of water a feeding tube should be returned to the pt.

2. flush the tube with 50ml of NS Pts with SIADH are usually on a strict fluid restriction to correct
water overload; therefore ALL FLUIDS (including enteral feeding
3. count the flush but not the feeding in planning fluid limitation and flush solution) should be considered when planning the fluid
restriction.
4. flush the tube with 50ml water to maintain patency
Because of the need for increased sodium, this pt should NOT
have the feeding tube flushed with water
ANSWER: 2,3,5
A nurse is assessing a pt with a tentative DX of hyperpituitarism.
Which assessment findings should the nurse observe for in this The pt with hyperpituitarism will exhibit the following:
pt? SELECT ALL THAT APPLY
tall stature if onset in childhood
1. short stature if onset is in childhood large hands and feet with prominent jawbone
joint changes consistent with arthritis
2. large hands and feet with prominent jawbone DEEP voice
dyshphagia
3. joint changes consistent with arthritis hypertension (HTN)
organomegaly
4. soft, high pitched voice skin changes leading to rough, oily texture

5. HTN The pt would NOT have a SOFT VOICE or be SHORT IN


STATURE

3/3

You might also like